Sunteți pe pagina 1din 59

1. P vs. Roxas, 58 Phil.

733, March 19, 1990

People v. Gonzales / G.R. No. 80762 / March 19, 1990

FACTS: Fausta Gonzales, Augusto Gonzales, Custodio Gonzales Sr., Custodio Gonzales, Jr., Nerio Gonzales
and Rogelio Lanida were all found guilty of murder for killing Lloyd Peacerrada. All of them except for
Custodio Sr. withdrew their appeal.The decision of the trial court was based on the testimony of witness
Jose Huntoria. Huntoria said that appellant was also one of the attackers of the deceased. Appellant
maintained that he was asleep at the moment. The trial court and the Court of Appeals rejected
appellants defense of alibi.

At around 9:00 o'clock in the evening of February 21, 1981, Bartolome Paja, the barangay captain of
Barangay Tipacla, Ajuy, Iloilo, was awakened from his sleep by the spouses Augusto and Fausta
Gonzales. Augusto informed Paja that his wife had just killed their landlord, Lloyd Peacerrada, and thus
would like to surrender to the authorities, Seeing Augusto still holding the knife allegedly used in the killing
and Fausta with her dress smeared with blood, Paja immediately ordered a nephew of his to take the
spouses to the police authorities at the Municipal Hall in Poblacion, Ajuy. As instructed, Paja's nephew
brought the Gonzales spouses, who "backrode" on his motorcycle, to the municipal building.[7] Upon
reaching the Ajuy Police sub-station, the couple informed the police on duty of the incident. That same
night, Patrolman Salvador Centeno of the Ajuy Police-Force and the Gonzales spouses went back to
Barangay Tipacla. Reaching Barangay Tipacla, the group went to Paja's residence where Fausta was
made to stay, while Paja, Patrolman Centeno, and Augusto proceeded to the latter's residence at Sitio
Nabitasan where the killing incident allegedly occurred.[8] There they saw the lifeless body of Lloyd
Peacerrada, clad only in an underwear, sprawled face down inside the bedroom.[9] The group stayed
for about an hour during which time Patrolman Centeno inspected the scene and started to make a rough
sketch thereof and the immediate surroundings.[10] The next day, February 22, 1981, at around 7:00 o'clock
in the morning, Patrolman Centeno, accompanied by a photographer, went back to the scene of the
killing to conduct further investigations. Fausta Gonzales, on the other hand, was brought back that same
day by Barangay Captain Paja to the police sub-station in Ajuy. When Patrolman Centeno and his
companion arrived at Sitio Nabitasan, two members of the 321st P.C. Company stationed in Sara, Iloilo,
who had likewise been informed of the incident, were already there conducting their own
investigation. Patrolman Centeno continued with his sketch; photographs of the scene were likewise
taken. The body of the victim was then brought to the Municipal Hall of Ajuy for autopsy.

ISSUE: Whether appellant was guilty.

RULING: The Supreme Court found Huntoria to be an unreliable witness. Huntoria admitted during cross-
examination that he cannot determine the group of people stabbing the deceased. He failed to
point definitely that appellant also did the crime. As stated in Arts. 3 and 4 of the Revised Penal Code, for
one to be criminally liable, an act should be committed. The SC found no sufficient proof that appellant
has acted. Lastly, the SC found Huntoria was an interested witness as he was also the tenant of the
deceased. His testimony was sought to ingratiate himself with the deceaseds family. The SC found
appellants guilt not proven by reasonable doubt thus acquitting him

.
Except Fausta who admitted killing Lloyd Peacerrada in defense of her honor as the deceased
attempted to rape her, all the accused denied participation in the crime. The herein accused-appellant,
Custodio Gonzales, Sr., claimed that he was asleep[30] in his house which was located some one kilometer
away from the scene of the crime[31] when the incident happened. He asserted that he only came to
know of it after his grandchildren by Augusto and Fausta Gonzales went to his house that night of February

1
21, 1981 to inform him.[32]

The trial court disregarded the version of the defense; it believed the testimony of Huntoria.

At this juncture, it may be relevant to remind that under our socio-economic set-up, a tenant owes the
very source of his livelihood, if not existence itself, from his landlord who provides him with the land to till. In
this milieu, tenants like Huntoria are naturally beholden to their landlords and seek ways and means to
ingratiate themselves with the latter. In this instance, volunteering his services as a purported eyewitness
and providing that material testimony which would lead to the conviction of the entire family of Augusto
Gonzales whose wife, Fausta, has confessed to the killing of Lloyd Penacerrada, would, in a perverted
sense, be a way by which Huntoria sought to ingratiate himself with the surviving family of his deceased
landlord. This is especially so because the need to get into the good graces of his landlord's family
assumed a greater urgency considering that he ceased to be employed as early as May
1981.[47] Volunteering his services would alleviate the financial distress he was in. And Huntoria proved quite
sagacious in his choice of action for shortly after he volunteered and presented himself to the victim's
widow, he was taken under the protective wings of the victim's uncle, one Dr. Biclar, who gave him
employment and provided lodging for his family.[48] Given all the foregoing circumstances, we can not
help but dismiss Huntoria as an unreliable witness, to say the least.

At any rate, there is another reason why we find the alleged participation of the appellant in the killing of
Lloyd Peacerrada doubtful--it is contrary to our customs and traditions. Under the Filipino family tradition
and culture, aging parents are sheltered and insulated by their adult children from any possible physical
and emotional harm. It is therefore improbable for the other accused who are much younger and at the
prime of their manhood, to summon the aid or allow the participation of their 65-year old[49] father, the
appellant, in the killing of their lone adversary, granting that the victim was indeed an adversary. And
considering that the appellant's residence was about one kilometer from the scene of the crime, [50] we
seriously doubt that the appellant went there just for the purpose of aiding his three robust male sons
(Custodio, Jr., Nerio, and Augusto), not to mention the brother and sister, Rogelio and Fausta, in the killing
of Lloyd Peacerrada, even if the latter were a perceived enemy.

Whether or not the client, under the evidence presented, has committed the felony of murder.

Held: No, he has not.

Ratio

Courts analysis of the evidence:

Investigation conducted left much to be desired. Centeno gave the date of commission as March 21, 1981.
The sketch made was troubling, as it did not effectively indicate the extent of the blood stains in the scenes
of crime. This would have added a lot of weight to any oneof the versions of the incident.

Sazon, who claimed that Gonzales surrendered to him, failed to state clearly the reason forthe surrender.
It may even be possible that Augusto surrendered just so he could be safe from the victims kin. Sazon also
admitted that Augusto never mentioned to him the

participation of other persons in the killing.-

2
Rojas statement showed two possibilities for the killing. Faustas admission that she was the only killer is
plausible. Furthermore, there were only five fatal wounds, which will be discussed later.-

Huntorias testimony, of which the prosecutions argument solely rests, needs to be examined further.
Huntorias claims in his testimony did not exact ly match with those from his cross-examination. He first
claimed that he recognized the people involved. However, in the cross- examination, he only saw
flashes.

This implies that he may not have recognized anyone at all.

As such, Huntorias testimony could not place a definite act committed or contributed by the appellant in
the killing of the deceased. On the criminal liability of the appellant:-

There is nothing in the findings or the evidence that establishes the criminal liability of the appellant as a
principal for direct participation under Art. 17, para. 1 of the Revised Penal Code.-

Furthermore, there is nothing in the findings or evidence that inculpates him by inducement, under
paragraph 2 of the same article. Based on the definition of felonies in Art. 3 of the Revised Penal Code, the
prosecutions evidence could not establish intent nor fault. Recall that the elements of felonies include: An
act or omission Act or omission must be punishable

Act is performed or omission incurred by deceit or fault-

The lone witness could not properly establish any acts or omissions done by the appellant. He stated that
he does not know who hacked or stabbed the victim, thus implying that he does not know what the
appellant did. With this, the essential elements of felonies may not even be present.-

Furthermore, the fact that there were five stab wounds and six accused would imply that one of them may
not have caused a grave wound (especially given the statement of thephysician). This may have been
the appellant, and given that there is no evidence that the appellant caused any of the wounds, coupled
with the prosecutions failure to prove the presence of conspiracy (that is, how many people actually took
part in the killing), it weakens the arguments against the appellant.On the lone witness:-

Huntorias credibility as a witness is tarnished by two points:

O He came out eight months after the killing. He claims that he feared for his life, but there was no proof
that he was being threatened, nor was the length of time reasonable given the circumstances.

O He is not exactly a disinterested/neutral witness. He admitted to being a tenant of the deceased, and
stated that one of the reasons why he testified was because the victim was his landlord.-

Under our socioeconomic set-up, a tenant owes the source of his livelihood from his landlord. As such, they
would do everything to get the landlords to their favour. Posing as a witness would have been a convenient
way to do this, especially as he ceased to be employed as early as May 1981.Finally, based on Philippine
customs and traditions, it is unlikely for the appellant to be in the scene of the crime, as under our family
culture, aging parents are usually sheltered and insulated from possible harm. It is improbable for the
accused to bring their aging father when they were clearly in better shape than he was, and it was unlikely

3
for the appellant to offer his services as they were more or less enough to handle what could have been a
perceived enemy. Although alibi is a weak defense, in cases like this where the participation of the
appellant is not clear, it may be considered. In light of the evidence on record, it may be sufficient for an
acquittal.

2. People vs. Alconga

Facts: On May 27, deceased Silverio Barion, the banker of the card game, was playing black jack against
Maria De Raposo. De Raposo and Alconga were partners in the game, they had one money. Alconga
was seated behind Barion and he gave signs to De Raposo. Barion, who was suffering losses in the game,
found this out and he expressed his anger at Alconga. The two almost fought outright this was stopped.

The two met again on May 29. when Alconga was doing his job as a home guard. While the said accused
was seated on a bench in the guardhouse, Barion came along and said Coroy, this is your breakfast
followed by a swing of his pingahan, a bamboo stick. Alconga avoided the blow by falling to the ground
under the bench with the intention to crawl out of the guardhouse. A second blow was given by Barion
but failed to hit the accused, hitting the bench instead. Alconga managed to go out of the guardhouse
by crawling on his abdomen. While Barion was about to deliver the 3rd blow, Alconga fired at him with his
revolver, causing him to stagger and hit the ground. The deceased stood up, drew forth his dagger and
directed a blow to the accused who was able to parry the attack using his bolo. A hand to handfight
ensued. The deceased, looking already beaten and having sustained several wounds ran away. He was
followed by the accused and was overtaken after 200 meters.

A second fight took place and the deceased received a mortal bolo blow, the one which slasehde
the cranium. The deceased fell face downward besides many other blows delivered. Alconga
surrendered.

Issue: Whether or not self-defense can be used as a defense by Alconga

Held: No. Self-defense cannot be sustained. Alconga guilty of Homicide

The deceased ran and fled w/o having to inflicted so much a scratch to Alconga, but after, upon the
other hand, having been wounded with one revolver shot and several bolo slashes the right of Alconga to
inflict injury upon him has ceased absolutely/ Alconga had no right to pursue, no right to kill or injure. He
could have only attacked if there was reason to believe that he is still not safe. In the case at bar, it is
apparent that it is Alconga who is the superior fighter and his safety was already secured after the first fight

4
ended. There was no more reason for him to further chase Barion. The second fight will be treated
differently and independently. Under the first fight, self-defense would have been valid, but that is not the
case in the second fight. In the second fight, there was illegal aggression on the part of Alconga and as a
result, he is found guilty of Homicide with no mitigating circumstance (MC) of Provocation

Note Provocation in order to be an MC must be sufficient and immediately preceding the act. It should
be proportionate to the act committed and adequate to stir one to its commission

Unlawful aggression no longer exists when the aggressor runs away after the attack (People vs.Alconga,
78 Phil. 366)

People v. Alconga

Facts:

On the night of May 27, 1943, in Mauricio Jepes' house in San


Dionisio, people were playing prohibited games. The deceased
Silverio Barion, the banker of black jack, was loosing to Maria de
Raposo because the accused Dioscoro Alconga was helping her by
communicating the cards of the deceased. Upon discovery of the
cheating, the deceased became angry and there was an exchange of
words between him and the accused. They almost hit each other if not
for the intervention of the maintainer. The deceased left,
threatening the accused, "tomorrow morning I will give you a
breakfast."

The deceased and the accused did not meet the next day. When the
accused was performing his duties as "home guard", the deceased came
along and, addressing the former, said, "Coroy, this is your
breakfast," then he tried to hit him. The accused avoided the blow
by crawling under the bench with the intention to runaway. He was
also avoided the second blow. The accused then managed to go out of
the guardhouse by crawling on his abdomen. While the deceased was in
the act delivering the third blow, the accused, while still in a
crawling position, fired at him with his revolver, causing him to
stagger and to fall to the ground. The deceased drew his dagger and

5
directed a blow at the accused, however, was able to dodge it with
his bolo. Then, there was a hand-to-hand fight. Having sustained
several wounds, the deceased ran away but was followed by the
accused. After running a distance of about 200 meters, the deceased
was overtaken, and another fight took place, during which the mortal
bolo blow caused the deceased to fall to the ground. The other
accused, Adolfo Bracamonte, arrived and, being the leader of
the "home guards" of San Dionisio, placed under his custody the
accused Alconga with the contention of surrendering him.

On their way to San Dionisio, the two accused were stopped by Juan
Collado, a guerrilla soldier. Adolfo Bracamonte turned over Alconga
to Collado who in turn took him to the headquarters. In the
afternoon of the same day, Collado delivered Alconga to Gregorio
Barredo, a municipal policeman of San Dionisio, together with the
weapons used in the fight: a revolver, a bolo, and a dagger.

Issue:

Whether or not the accused, Alconga, can invoke self-defense.

Ruling:

Alconga was found guilty of the crime of HOMICIDE.

Reason:

There were two stages in the fight:


First - commenced when the deceased assaulted appellant without
sufficient provocation on the part of the latter There was
unlawful aggression because he was being attacked. There was
employed reasonable necessity of means used to prevent it because,
as a security guard, he has in hand a revolver. There was no
sufficient provocation on his part.

Second stage when the deceased ran and was pursued by the accused.

6
When the deceased ran, it meant that the unlawful aggression
subsisted.

Can we find under the evidence of record that after the cessation of said aggression the provocation thus
involved therein still persisted, and to a degree sufficient to extenuate appellant's criminal responsibility for
his acts during the second stage of the fight? Appellant did not testify nor offer other evidence to show
that when he pursued the deceased he was still acting under the impulse of the effects of that
provocation, be it anger, obfuscation or the like. The Revised Penal Code provides:
"ART. 13. Mitigating circumstances:
* * * * * * *
"4. That sufficient provocation or threat on the part of the offended party immediately preceded the act."
It is therefore apparent that the Code requires for provocation to be such a mitigating circumstance, that
it not only immediately precede the act but that it also be sufficient. In the Spanish Penal Code, the
adjective modifying said noun is "adecuada" and the Supreme Court of Spain in its judgment of June 27,
1883, interpreted the equivalent provision of the Penal Code of that country, which was the source of our
own existing Revised Penal Code, that "adecuada" means proportionate to the damage caused by the
act.
Justice Albert, in his commentaries on the Revised Penal Code, 1946 edition, page 94, says: "The
provocation or threat must be sufficient, which means that it should be proportionate to the act committed
and adequate to stir one to its commission" (italics supplied).
Sufficient provocation, being a matter of defense, should, like any other, be affirmatively proven by the
accused. This the instant appellant has utterly failed to do. Any way, it would seem self-evident that
appellant could never have succeeded in showing that whatever remained of the effects of the
deceased's aggression, by way of provocation after the latter was already in flight, was proportionate to
his killing his already defeated adversary.
3. People vs. Gandayao

4. G.R. No. L-28129 October 31, 1969ELIAS VALCORZA,


petitioner, vs.
PEOPLE OF THE PHILIPPINES, respondent.
RULING
The petitioner is acquitted.

ART. 11. Justifying circumstances. - The following do not incur any criminal liabilityParagraph 5. Any
person who acts in the fulfillment of a duty or in the lawful exercise of a right or office.The facts and
circumstances constrain us to hold that the act thus performed by petitioner,which unfortunately
resulted in the death of the escaping detainee, was committed in the performance of his official
duty and was more or less necessary to prevent the escaping prisoner from successfully eluding the
officers of the law. To hold him guilty of homicidemay have the effect of demoralizing police officers
discharging official functions identical or similar to those in the performance of which petitioner was
engaged at the time he fired at the deceased Pimentel, with the result that thereafter We would
have half-hearted and dispirited efforts on their part to comply with such official duty. This of course,
would be to the great detriment of public interest.FACTS:The deceased, Roberto Pimentel was a

7
detention prisoner who escaped. While in search for the escaped prisoner, Sgt. Daiton saw a person
approaching slowly under the bridge and he ordered him to halt. The latter instead of doing so,
jumped down into the creek spanned by the bridge. Roberto Pimentel emerged suddenly from the
bushes and lunged at the appellant Valcorza, hitting him with a stone and causing him to fall to the
ground. Appellant Valcorza regained his composure and immediately chased the deceased, as
the deceased did not heed his order to stop, appellant fired four times into the air, and a fifth shot
at Pimentel as the latter was in an act of again jumping down into another part of the creek. The
members of the patrol team went down into the water to locate Pimentel and they saw him floating,
with a wound on his back. Elias Valcorza surrendered himself and his firearm to the Chief of Police

ISSUE: Whether or not peace officer Valcorza was justified in shooting the deceased

Give an example of Article 13, par 4.

5. PEOPLE v MANANSALA, Jr.,


31 SCRA 401
G.R. No. L-23514
Feb. 17, 1970

Facts:
Early in the afternoon of March 27, 1962 Avelino was eating his lunch inside one of the
eateries dotting the market site, when Rodrigo Aringo alias Diego, a baggage boy in the same
market, approached him and demanded his fee for having carried Avelino's baggage. Avelino
said he was willing to pay for the services rendered at noon, but not for those rendered earlier in the
morning. He then took some money from his pocket and proffered it to Rodrigo. Obviously peeved
at having been thus publicly rebuffed, Rodrigo brusquely brushed Avelino's hand aside and instantly
gave him a fist blow in the face. Avelino fell from his seat; he tried to get up, but was given another
blow, and then a third. As he reeled from the force of the last blow a batangas knife he was carrying
fell from his trousers pocket. Avelino picked up the knife, and Rodrigo, seeing that he was armed,
rushed to the carinderia's kitchen and returned almost immediately with a 10-inch knife in his hand.
With it he swung at his antagonist, but the latter evaded the blow.

Meanwhile, appellant Jose Manansala, an uncle of Avelino, noticed the commotion from
outside the carinderia, where he was loading baggage on a parked truck some six meters away.
He shouted at Rodrigo to stop. Rodrigo paid no heed and instead delivered another thrust at
Avelino, who again evaded it. At the third attempt, Jose embraced Rodrigo from behind, and it
was at that moment that Avelino stabbed Rodrigo and inflicted the numerous wounds which
proved fatal. Jose took the kitchen knife from Rodrigo and threw it away, and then released his
limp body on the papag nearby. When the policemen arrived Avelino was still clutching the knife
he had used. Both appellants were apprehended.

Issue:
Whether or not there was sufficient provocation on the part of the offended party.
Held:
Yes. While it is clear that Avelino did not act in legitimate self-defense, the trial court correctly
held that there was sufficient provocation on the part of the victim. The evidence given by the

8
witnesses for the defense as to how and why the fight started, and as to the fact that the deceased
hit Avelino with his fist, is not controverted by the witnesses for the prosecution, who did not see the
incident from the very beginning. And one fist blow at least is confirmed by the doctor who treated
Avelino for a contusion around one eye.

6. [G.R. No. L-32390. December 28, 1973.]

THE PEOPLE OF THE PHILIPPINES, Plaintiff-Appellee, v. CLEMENTE AQUINO, Defendant-Appellant.

Solicitor General Antonio P. Barredo, Assistant Solicitor General Crispin V . Bautista and Solicitor
Enrique M. Reyes for Plaintiff-Appellee.

Feliberto V . Castillo, for Defendant-Appellant.

DECISION

In Criminal Case No. 6404-M of the Court of First Instance of Bulacan, Clemente Aquino was charged
with and found guilty of murder for the death of Pedro Cruz in the afternoon of April 29, 1967 at barrio
Sibul, San Miguel, Bulacan. He was sentenced to reclusion perpetua, to pay an indemnity of P12,000.00
and P10,000.00 for actual expenses and moral damages suffered by the heirs of the victim. The case
was thereafter elevated to this Court on appeal.

There is no dispute as to the fact of the killing Aquino did it with a revolver, caliber .22, which he fired
four times in succession. His plea is self-defense.

Cruz was an employee of the Farmont Mines and Aquino was working as truck driver for a certain
Leoncio de Guzman, owner of a white clay mine adjacent to the Farmont property. It seems that there
was some king of rivalry between the two mining establishments, which somehow involved their
respective personnel.

The case for the prosecution consists of the testimony of three principal witnesses, all employees of the
Farmont Mines namely, Priscila Corpuz, Jorge Cruz and Dominador Cruz. In the morning of April 29, 1967
they rode together with Pedro Cruz in the latters jeep to go to the Farmont Mines site at Akle, San
Ildefonso, Bulacan. On their way they passed Bo. Pinaod, where they saw a truck parked by the
roadside and a jeep just behind. Clemente Aquino was at the drivers seat of the truck and Leoncio
de Guzman was inside the jeep. Priscila Corpuz said she afterwards noticed the two vehicles following
her group at a distance of about 100 meters. A half hour after she and her companions reached the
mine site Clemente Aquino arrived in his truck and "passed thru the fence" of the Farmont Mines. Pedro
Cruz was then inside the Farmont property, repairing his own jeep, and after Aquino rammed the truck
through the fence, Cruz went directly to his jeep and drove it to intercept the other vehicle. When Cruz
caught-up he alighted from the jeep and without any previous exchange of words between them
Aquino immediately shot him.

Jorge Cruz related substantially the same story as that given by Priscila Corpuz, except that while the
latter said she joined her companions before reaching Bo. Pinaod, where she got off the jeep for a
while to buy some foodstuffs, Jorge Cruz said that it was in the said barrio that they picked up Priscila,
he himself having joined Pedro Cruz in the latters jeep much earlier in the day.

The third witness for the prosecution was Dominador Cruz, who was also in the group which rode in the
jeep of the deceased Pedro Cruz in going to the mine site. The version of all three is that after alighting

9
from his jeep, which he used to intercept Clemente Aquinos truck, catching up with the latter on the
dry bed of the river near the Farmont property, Pedro Cruz walked to where the truck had stopped. He
was standing some two or three meters from the truck, to the left of the drivers seat, when Clemente
Aquino shot him four times, killing him on the spot.

Aquinos version as to the circumstances which led to the shooting contradicts that of the prosecution.
He did not drive thru or ram the fence of the Farmont Mines. He was a truck driver for Leoncio de
Guzman, his shift of duty being in the afternoon, while another driver, Melencio Guevarra, drove the
truck in the morning. At 11:30 on April 29, 1967, he took over as driver at the clay stockpile in Bo. Akle
and left for the mine site, together with Guevarra and Teofilo de Guzman. On the way he gave a lift to
several persons, including an army soldier by the name of Conrado Ferma, the latters wife and two
children. To go to the mine site he did not have to pass thru the Farmont property because some
distance from it a passageway to the dry river bed had been constructed by Leoncio de Guzmans
laborers. While he was nogotiating that passageway, however, he accidentally rammed three or four
bamboo posts which were stuck into the ground. He proceeded on the dry river bed, stopped to
unload the soldier and his family near their hut, and then continued on his way. He had covered a short
distance when he saw a jeep speeding in his direction, with Pedro Cruz at the wheel and a companion,
called "Moro," beside him. Cruz blocked his way with the jeep, got off and walked towards the left side
of the truck. About two meters away Cruz stopped and asked him, "Why do you insist passing here
when I am stopping you?" Aquino answered, "Why should you stop me here when I am already on the
river?" Cruz did not answer, but gave a meaningful dance at "Moro," who was still inside the jeep, and
touched something bulging at his waist. When he pulled the thing out, Aquino saw it was a "balisong"
knife. Cruz stepped on the left running board of the truck, and hauled himself up with his left hand
holding on to the windshield frame. Aquino moved towards the right side of the drivers seat but was
not able to go to the extreme end because his thigh got wedged under the gear shift. Cruz thrust
forward with his knife at him, but he leaned far to the right and at the same time parried the knife hand.
Cruz switched his knife to a stabbing position and at that moment Aquino, who was already leaning
almost prone on the drivers seat, got his gun from the tool box and fired at his antagonist. The first shot
hit Cruz on the right side of the throat, the second on the right arm. Aquino fired four times in all and
then jumped out of the truck on the right side, leaving Cruz slumped over the drivers seat. Aquino
sought cover near the right front tire of the truck. "Moro," gun in hand, got off the jeep but ran away
when he saw Aquinos gun trained on him. When Aquino again looked in the direction of his antagonist,
he saw the latter already lying beneath the truck. There was a small army contingent nearby and he
went to one of the huts of the soldiers and surrendered his gun to one of them.

The appellants version was corroborated by Teofilo de Guzman, former employee of Leoncio de
Guzman. He was one of those who rode with Aquino from Leoncio de Guzmans stockpile of white clay
in Bo. Akle. Besides himself there were other persons who were given a lift in the same truck, including
an army soldier with his wife and two children. They drove on a public road, and upon reaching a spot
where there was a signboard marked "Farmont Property," Aquino made a detour toward Akle river. In
negotiating the bend down to the river bed, the truck accidentally rammed and destroyed four
bamboo posts. They stopped briefly to allow the soldier and his family to get off. Suddenly a jeep, which
was running very fast, overtook them and stopped in front of the truck. Pedro Cruz, who was at the
wheel, got down, leaving his companion named "Moro" inside. Cruz walked toward the truck and there
was an exchange of words between him and Aquino. He pulled out a bladed weapon, stepped on
the running board of the truck and lunged at his opponent, who evaded the attack by leaning down
to his right. Both Aquino and Teofilo de Guzman testified that the blood from the bullet wounds Pedro
Cruz received spilled on the drivers seat of the truck. De Guzman also testified that during the shooting
the other passengers who were still inside the truck jumped out and that he saw Cruz slide down slowly
from the running board, with his two legs finally going directly under the truck.

10
In view of the conflicting versions of the witnesses for the prosecution on one hand and the witnesses
for the defense on the other, a number of physical and objective circumstances assume decisive
importance.

First is the testimony of patrolman Ernesto Salazar of the San Miguel police. He was listed in the
information as one of the witnesses of the prosecution but was not called to testify although he was
present in court all the time that the State was adducing its evidence. Presented instead by the
defense, he said that in the afternoon of April 29, 1967, a policeman from San Ildefonso reported the
death of Pedro Cruz to the Chief of Police of San Miguel. The Chief then ordered him and patrolman
Sta. Maria to proceed to the scene of the incident. He could not recall how many persons were there
when they arrived, but did remember a certain Victor Intacto and several army soldiers, headed by
Sgt. Oscariz, standing near the dead body. It was then about 8 oclock in the evening. He saw Pedro
Cruz lying on his back by the left side of the truck, and had photographs of the body taken from
different angles; he found blood stains on the drivers seat and had them also photographed; and then
afterwards he found a dagger on the floor of the truck near the drivers seat, the scabbard of which
he had previously seen tucked at Pedro Cruz right hip. In the course of his testimony Salazar identified
the eight photographs which had been taken, his on-the-spot report and the sketched of the scene
which he prepared, the dagger and the scabbard. In particular the photographs of the body show
that where he lay Pedro Cruz two legs were directly under the truck.

Another significant piece of evidence is the biology report and testimony of Mercedes Bautista, Chief
of Forensic Chemistry of the National Bureau of Investigation, to the effect that she analyzed the blood
stains on the drivers seat of the truck in question upon request by the Chief of Police of San Miguel and
found them to be of human blood.

The physical, objective facts enumerated above are not only consistent with but indeed confirm
strongly the plea of self-defense raised by the appellant. The direction of three of the four bullets which
hit the deceased shows that he must have been in a forward stooping position at the time, with his left
forearm raised somewhat in front of him, as would be the case if he was holding the windshield frame
with his left hand. Thus the slug which entered the base of the neck, in front and to the right, plowed
downward through the upper lung and the muscle of the lumbar region, where it was recovered just
beneath the skin. The slug which entered the body at the left upper chest also followed a downward
direction and was recovered beneath the skin at the back. And the slug which hit the left forearm near
the palm of the hand took both an upward and posteriorly direction and exited "at the proximal 3rd of
the forearm medially." The bullets could not have had these trajectories if the deceased had been
standing upright two or three meters to the left of the truck, as the witnesses for the prosecution testified.

The evidence of the dagger or knife, which was retrieved by patrolman Salazar from the floor of the
truck below the drivers seat, and the evidence of the blood stains on the seat itself, not only find no
explanation in but directly contradict the version of the prosecution. It should be noted that although
the chemical examination of the stains was done eleven days after the incident, they were
photographed shortly after it happened and the drivers seat itself was placed in the custody of the
chief of Police of San Miguel.

The trial court took a cavalier attitude toward these evidences, hinting that they might have been
planted But the one who found the knife and had photographs of the blood stains taken was the police
officer assigned to conduct an official investigation. And he did 90 in the presence of several persons.
The appellant himself surrendered to an army soldier and was taken away immediately after the
shooting; and the premises where it happened, including the truck and the body of the deceased,
were under guard by army soldiers before patrolman Salazar arrived to conduct the investigation. The
very position of the body as testified to by the same patrolman and confirmed by the photographs

11
shows that after being shot Cruz must have slid down the running board of the truck where he was said
to be standing, thus accounting for the fact that he was found lying on his back with his two legs under
the truck. Such a position could hardly have been possible if he had been shot while standing two or
three meters away from the vehicle.

The three elements of self-defense are here present. There was unlawful aggression on the part of the
deceased when he attacked the appellant with a deadly weapon, especially in the rather cramped
quarters of the drivers compartment of the truck, where the space was too limited for effective
maneuvering. The means employed to repel that aggression was reasonable. As repeatedly held by
this Court," (I)n emergencies of this kind, human nature does not act upon process of formal reason but
in obedience to the instinct of self-preservation; and when it is apparent that a person has reasonably
acted upon his instinct, it is the duty of the courts to sanction the act and hold the act irresponsible in
law for the consequences." 1 And upon a review of the evidence we find that the appellant did not
give any sufficient provocation for the attack against him.

In ruling that there was such provocation the court a quo relied on the testimony of Priscila Corpuz to
the effect that the appellant rammed the closed gate of the Farmont compound. However, this
testimony is of doubtful credibility: Corpuz gave conflicting versions on this point which she failed to
explain satisfactorily. In her affidavit (Exhibit 4) which she executed before a certain Atty. Ernesto
Catimbang, she stated:jgc:chanrobles.com.ph

"6. That we saw the actual shooting incident, when without any warning the accused driver of the
cargo truck upon arriving at the gate ordered Mr. Pedro Cruz to move his jeep away from the road in
order to pass by and the deceased Pedro Cruz answered that the truck can pass because he is parked
by the side of the road but instead of passing by the cargo truck driver suddenly and without warning
pulled a gun hidden in the compartment, of the truck and shot at Pedro Cruz, who because he was hit
had to climb the truck but was repulsed by the succeeding shot fired at him and fell to the ground
dead."cralaw virtua1aw library

In her two versions quoted above Priscila Corpuz did not say that the appellant forced his way through
the closed gate of the Farmont Mines or that he went inside its premises in going down the river bed.

On the other hand, the appellant clearly testified, with the aid of sketch (Exhibit 1) and with
corroboration from Teofilo de Guzman, that before reaching the gate of the Farmont Mines he made
a detour to the river bed by using the passageway which had been constructed by the men of Leoncio
de Guzman and that while negotiating the descent he accidentally bumbed three or four bamboo
posts stuck into the ground. The existence of the passageway, which according to the defense was
constructed purposely to avoid passing through the contested road, was not controverted by the
prosecution.

In view of the foregoing considerations We find the plea of complete self-defense sufficiently
established. The judgment of the Court a quo is reversed and the appellant is acquitted, with costs de
oficio.

7. People vs. Encomeinda

8. People vs. Jaurigue

12
Facts: Inside the chapel of the 7th day Adventist Church, AmadoCapina sat beside the appellant and
with the greatest of impudence, placed his hand on the upper part of her right thigh. On observing this
highly improper and offensive conduct, Avelina Jaurigue, conscious of her personal dignity and honor,
pulled out a fan knife and stabbed Amado once at the base of the left side of the neck

Amado Capina died from the wound a few minutes later.

Appellant Avelina Jaurigue was subsequently tried and convicted of the crime of Homicide.

Issue: Whether or not appellant Jaurige acted in the legitimatedefense of her honor and that she should
be completely absolved of all criminal responsibility

Held: No. The judgment of conviction is affirmed.

The attempt to rape a woman constitutes an unlawful aggression sufficient to put her in a state
of legitimate defense inasmuch as a woman's honor cannot but be esteemed as a right as precious, if not
more than her very existence; and it is evident that a woman who, thus imperiled, wounds, nay kills the
offender, should be afforded exemption from criminal liability, since such killing cannot be considered a
crime from the moment it became the only means left for her to protect her honor from so great an
outrage (1 Viada,Codigo Penal, 5th ed., p. 301; People vs. Luague and Alcansare, 62 Phil., 504). .

As long as there is actual danger of being raped, a woman is justified in killing her aggressor in the defense
of her honor.

When the deceased sat by the side of the appellant on the same bench, near the door of the barrio
chapel and placed his hand on the upper portion of her right thigh without her consent, the said chapel
was lighted with electric lights, and there were already several people inside the chapel, including her
own father and the barrio lieutenant and other dignitaries of the organization; and under
thecircumstances, there was and there could be no possibility of her being raped. And when she
gave Amado Capina a thrust at his neck, inflicting upon him a mortal wound and causing his death a few
moments later, the means employed by her in the defense of her honor was evidently excessive; and
under the facts and circumstances of the case, she cannot be legally declared completely exempt from
criminal liability.

9. 9. People vs. Apolinar


10. 10. People vs. Sotelo

G.R. No. L-33304 December 13, 1930

THE PEOPLE OF THE PHILIPPINE ISLANDS, plaintiff-appellee,

vs.

CONSTANTE SOTELO, ET AL., defendants.

CONSTANTE SOTELO, appellant.

Alberto Reyes for appellant.

13
Attorney-General Jaranilla for appellee.

VILLAMOR, J.:

The Sotelo brothers, namely, Constante, Dominador, and Vicente, were prosecuted in the Court of First
Instance of Ilocos Sur for the crime of homicide under the following information:

That on or about the night of December 24, 1929, in the municipality of Narvacan, Province of Ilocos Sur,
Philippine Islands, the said accused Constante, Dominador, and Vicente Sotelo, armed with a penknife, a
stick, and an iron bar, respectively, acting together and helping one another, did willfully, maliciously,
unlawfully, and feloniously with treachery and evident premeditation attack, beat up, and commit assault
upon the person of Ignacio Cambaliza, inflicting a mortal wound upon him on the level of the left nipple,
which penetrated the left lung and the left ventricle of the heart, another on the outward surface of the
right arm, a bruise on the nose and another on the upper lip: as a result of which said Ignacio Cambaliza
died after a few minutes.

Contrary to law; with the aggravating circumstance of abuse of superior strength.

After the trial, the court below found the defendant Constante Sotelo guilty of the crime of homicide, and
the defendants Vicente and Dominador Sotelo of slight physical injuries, sentencing the former to suffer
twelve years and one day of reclusion temporal, to indemnify the heirs of the deceased in the sum P1,000,
with the accessories of law, and to pay one-third of the costs; and ordering the release of Vicente and
Dominador Sotelo in view of the fact that they had already been imprisoned since December 24, 1929,
with two-thirds of the costs de oficio.

The defendant Constante Sotelo appealed from this judgment, and his counsel has made the following
assignments of error:

I. The trial court erred in accepting the whole theory of the prosecution:

Despite the obvious incongruity between the information and the evidence adduced at the
trial;lawphi1>net

Despite the proof that the iron bar, Exhibit B of the prosecution and Exhibit 1 of the defense, belong to the
principal witness, Baltazar Capistrano, and not to any of the defendants;

14
Despite the fact that it has been proved that the incident took place in the yard of the defendants' house
and not on the public road;

Despite the fact that Baltazar Capistrano deliberately concealed the place where the deceased expired,
fearing his participation in the tragedy should come to light; and

Despite the fact that the chief of police acted with evident partiality towards Cambaliza and Capistrano,
in intervening immediately after the incident had occurred, hiding the whip or riding crop, which the other
local authorities found hanging from the right arm of the deceased, and for the other reasons.

II. The trial court erred in finding the following facts: that Dominador Sotelo hit the deceased across
the mouth with the crop Exhibit B, Vicente Sotelo stabbed him on the right shoulder with the penknife
Exhibit D, and Constante Sotelo stabbed him under the nipple.

III. The trial court erred in refusing to consider the plea of self-defense alleged and proved by the
defendant-appellant, Constante Sotelo, notwithstanding the fact that said defense is strongly
corroborated by Exhibits O, 5, and 4, which form a part of the res gestae.

IV. The lower court erred in refusing to consider the motion of February 1, 1930, declaring afterwards
that, as the defendant has shown graphically, the latter could not have touched the victim's left breast in
stabbing him, or the wound would not have been in the direction described in Dr. Nolasco's certificate; in
spite of the fact that it had made contrary declarations during the trial, which estop it from making the
subsequent holdings.

V. The trial judge erred in convicting the defendant appellant, Constante Sotelo.

The record shows that at about 8 o'clock in the evening of the 24th of December, 1929, Ignacio Cambaliza
started for the barrio of Ravadabia, in the municipality of Narvacan, Ilocos Sur, accompanied by Baltazar
Capistrano. They took the provincial road leading to said barrio and as they approached the Sotelo house,
they bid the time of day, asking whether they might pass by, according to the custom of the place. They
were barely 20 meters away from the house, when the defendant Constante Sotelo, who had just finished
his supper, descended from the house towards the road, and, from the entrance of his yard, turned his
flashlight on the passers-by to see who they were. When Ignacio Cambaliza saw this, he walked back to
where Constante Sotelo stood and inquired why he turned his flashlight on them, and what it was he
wanted, winding up with a vulgar remark. When Constante's brothers, who were then in the yard on the
side of the road, saw Cambaliza's attitude, they approached their brother to separate or defend him,
whereupon Cambaliza's commenced beating them with his iron crop, once striking Constante's arm. The
brothers, in turn, fell upon Cambaliza, Dominador striking him across the face with the stick he carried, and
Vicente wounding him in the right shoulder with a penknife. At this juncture, Capistrano attempted to
intervene, but he was warned by Vicente and probably by Dominador also, for which reason he withdrew

15
from the scene, and the fight then continued between Cambaliza and Constante. In the course of this
fight, Constante thrust a penknife into Cambaliza at about the level of the left nipple, producing a wound
which penetrated the left lung into the left ventricle of the heart, resulting in his death a few minute later.

Witness Baltazar Capistrano, who was with the deceased, says that after Ignacio Cambaliza had fallen
lifeless, he went over to the municipal building to ask for help, and at once the chief of police, the justice
of the peace, and the municipal president repaired to the place where the incident had occurred,
placing the brothers under arrest that same night: Constante with a penknife, Vicente with another
penknife, and Dominador with a cane.

Doctor Antonio Nolasco examined Ignacio Cambaliza's body, and found a knife wound at about the level
of the left nipple, 2 1/2 centimeters long, and 2 1/2 inches deep; a knife wound on the outward surface of
the right arm, 4 centimeters deep; and bruises at the base of the nose and on the upper lip, produced by
a blunt instrument. According to the doctor the breast wound which pierced the left lung and the left
ventricle of the heart was the cause of Ignacio Cambaliza's death.

The appellant admits he inflicted the injury which resulted in Ignacio Cambaliza's death, but maintains he
did so in self-defense. In support of this allegation it is insisted that the fight took place in the yard of the
defendants' house; that Vicente and Dominador went to help their brother Constante when they saw
Cambaliza attack him, but ran behind some sugar cane near by in order to conceal themselves when
pursued by Cambaliza; that the latter fought hand to hand with Constante, choked him and threw himself
upon him, and at that instant Constante thrust a penknife into his ribs below the left nipple; that Capistrano
answered Cambaliza's call, saying that Constante had stabbed him, and, with the assistance of
Capistrano, Cambaliza succeeded in leaving the Sotelo yard and after a few steps fell lifeless on the
roadside, where his body was later found.

After examining the evidence of record, we believe the defense is, in a measure, supported by the
testimony of Baltazar Capistrano given before the justice of the peace of Narvacan. In considering this
proof it is well to remember that the justice of the peace of Narvacan testified in the case that Baltazar
Capistrano made two statements before him in the investigation prior to the arrest of the defendants,
which is Exhibit 9, and in the course of the preliminary investigation, embodied in Exhibit 10. Counsel for the
defense attempted to examine Capistrano on these two statements, but the fiscal objected, and the court
sustained the objection on the ground that the best evidence would be Capistrano's own statements
taken down in writing. Counsel then required the fiscal to present said documents, and the latter delivered
to him the aforementioned Exhibits 9 and 10, which were offered in evidence by the defense. The fiscal
reiterated his objection to that evidence on the ground that the documents were not identified; but the
record shows that they had been delivered by the fiscal himself to counsel for the defense, and he is
therefore precluded from setting up the lack of identification, whereupon the court doubtless ruled them
in, and the fiscal failed to take exception therefrom.

The record further shows that counsel for the appellant sought to have the witness Capistrano explain the
contradiction between his statement in Exhibit 10 and his testimony at the hearing, but the fiscal objected

16
and the court sustained the objection. Capistrano has thus failed to explain the contradiction noted
between Exhibit 10 and his testimony before the trial court.

We believe the trial judge erred in sustaining the fiscal's objection to having witness Capistrano explain the
contradiction between his statement in Exhibit 10, and his testimony before the trial court at the hearing.
But be that as it may, we are of opinion that said documents Exhibits 9 and 10 have been duly introduced
into the case as evidence for the defense and must therefore be taken into consideration in rendering
judgment.

In said Exhibit 10, witness Capistrano, among other things, affirms the following:

As we passed by Constante was flashlighting us and he was standing by the door of their yard. I did not
see any body by him. We were then about 20 meters away from him when he rushed to us.

Ignacio was the first one who uttered bad words against Constante. Ignacio had a whip wrapped with
lead.

Other than this time I declared before the justice of the peace.

As Dominador came he struck Ignacio, but Ignacio defended. Vicente struck and Constante rushed in
and then they wrestled against each other and I tried to separate them.

I was only 5 meters away from them when they first wrestled. It was dark at that time but I saw what
happened by the aid of my flashlight.

I am very sure that Exhibit A was the bar which Vicente was holding and Exhibit B was the knife that
Constante used in stabbing Ignacio.

While they were wrestling Constante was under Ignacio during which time, I saw Constante bring out his
knife.

The same witness testified before the trial court as follows:

Q. Do you know whether anything extraordinary took place along the road? A. Yes sir; for when we
came near the house of Francisco Sotelo, we saw Constante, Dominador, and Vicente Sotelo in the yard

17
of the house. I and my companion said "we are passing, sir," but they did not answer, and they focused
their flashlight on us. And when we came to within ten meters, they still kept the flashlight focused on us,
and then my companion said: "Why do you turn your flashlight on us? Have we not greeted you in passing?"

Q. What did the Sotelo brothers, Constante, Dominador, and Vicente do when they heard Ignacio
Cambaliza say this? A. After Ignacio Cambaliza had said that, they switched off the light, and we
continued on.

Q. What else? A. After we had proceeded about 20 meters, someone came up behind us with a
flashlight saying: "Wait! your mother's . . .! you cannot say bad words when you pass here."

Q. What else? A. When they had said this, Ignacio Cambaliza stopped and said: "I have used no
bad word."

Ignacio Cambaliza interrupted Constante Sotelo saying "I did not say that." And Dominador, in turn, said,
"You didn't say anything, your mother's . . .!"

Q. And what did Dominador Sotelo do then? A. Immediately after saying, "you didn't say anything,
your mothers . . .! he struck him across the face with a stick.

xxx xxx xxx

Q. And what else happened? A. After that blow delivered by Dominador Sotelo, he was stunned.
While he was so stunned, Vicente Sotelo stabbed him with a penknife on the right shoulder; and Ignacio
Cambaliza, still stunned, turning about; and Constante Sotelo stabbed him in the region of the heart with
a penknife.

Comparing the two statements quoted above, it will be seen that the witness Capistrano affirmed in Exhibit
10 that he saw Constante standing at the entrance of his yard, alone; that the deceased was the first to
make vulgar remarks to Constante; and that during the fight Constante was under Cambaliza when he
drew his penknife to stab him. On the other hand, testifying before the trial court, the said witness
Capistrano stated that he and the deceased saw the three brothers, Constante, Dominador, and Vicente
Sotelo in the yard of the house; that one of these brothers was the first to make insulting remarks to
Cambaliza, and that while the two brothers were attacking Cambaliza, Constante stabbed him in the
chest with a penknife.

We believe Capistrano's testimony appearing in Exhibit 10 as to Constante's position when he wounded


Cambaliza, must be accepted, not only because it was given two days after the incident, but because it

18
has been corroborated by the witnesses for the defense. (U. S. vs. Capisonda, 1 Phil., 575; and U. S. vs.
Rafael, 23 Phil., 184.)

As to who started the aggression, there is an obvious contradiction between the testimony of Capistrano
and that of the witnesses for the defense. But in the light of sound judgment, we are inclined to believe
that the deceased started the aggression, provoked by the offensive language used by Constante and
his brothers, imputing to him the utterance of vulgar language against them. In such a situation the
deceased naturally used his whip against those who were in front of him, striking Constante's arm. We
therefore believe this is a case of incomplete self-defense, wherein the appellant was unlawfully attacked
by the deceased and compelled to employ reasonable means to defend himself, but he is responsible for
provoking the attack. (U. S. vs. Ancheta, 1 Phil., 30; U. S. vs. McCray, 2 Phil., 545.) According to article 86 of
the Penal Code the penalty next below that provided in article 404 of said Code must be imposed upon
the appellant, that is, prision mayor in its minimum degree, or six years and one day, with the accessories
of law, and to indemnify the family of the deceased in the amount of P500. And with this modification the
judgment appealed from is affirmed in all other respects, with costs against the appellant. So ordered.

11. People vs. Hernandez

PEOPLE v. NORMA HERNANDEZ (1959)


BY MAROON 5 PARTNERS AND ASSOCIATES JUNE 1, 2012 SLANDER
Plaintiff-Appellee: People of the Philippines

Defendant-Appellants: Maria Norma Hernandez, Mariano Hernandez (father) & Ramona Martinez (mother)

FACTS:

Vivencio Lascano, 19 y/o, started courting appellant, Maria Norma Hernandez and after months of courtship,
appellant finally accepted Vivencio. On the same date, she asked him to bring his parents over her home so
that they could talk about their marriage.
When Vivencio and his parents went to her house, they brought chickens and goats and they agreed to buy
a wedding dress, 2 vestidas, shoes, P20 for the sponsors and to repair the uncles roof.
While the celebration was going on, appellant was nowhere to be found. Vivencio and his parents waited
but she never showed up thus causing them great shame and humiliation.
Norma Hernandez averred that Vivencio was really courting her but that she wasnt really in love with him. Her
parents tried to persuade her to accept the proposal and that she only accepted it out of obedience to her
parents and the uncles insistence.
Before Vivencios parents came to their home, she already counselled them not to bring the chickens and
that they should not regret whatever may happen later.
Appellant said she felt torture because she wasnt honestly in love with Vivencio and so she decided to leave
home as last recourse to prevent the marriage.
Appellants parents also corroborated her testimony.
RTC convicted her of serious slander by deed because she purposely and deliberately fled to prevent
celebration of marriage. Thus, she appealed.

19
HELD:

Court reversed the RTC judgment and acquitted the appellant.

RATIO:

Malice, one of the essential requisites of slander hasnt been proven. There is no malice in the act of the
appellant changing her mind. She was merely exercising her right not to give her consent the marriage after
mature consideration.
Furthermore, there were no strained relations existing between the complainant & appellant before the
incident. There always existed good relations between them for they were neighbours so it cannot be
sustained that appellant was motivated by spite or ill-will in deliberately frustrating the marriage.
Appellant has the privilege to reconsider her previous commitment to marry and it would be utterly inconsistent
to convict her for slander by deed simply because she desisted in continuing with the marriage. If she would
be liable then that would be tantamount to compelling her to go into a marriage without her free consent.
Appellant had the right to avoid to herself the evil of going through a loveless marriage. (Art. 11 par.4, RPC)

PEOPLE V BERONILLA (12)

1FEB

L 4445 | February 28, 1955 | J. JBL Reyes


Obedience to Lawful Order of a Superior
Facts:
Manuel Beronilla, Policarpio Paculdo, Filipino Velasco and Jacinto Adriatico file an appeal from
the judgement of the Abra CFI, which convicted them of murder for the execution of Arsenio
Borjal, the elected mayor of La, Paz, Abra (at the outbreak of war), which was found to be aiding
the enemy.
Borjal moved to Bangued because of death threats was succeeded by Military Mayor Manuel
Beronilla, who was appointed by Lt. Col. Arbold, regimental commander of the 15 th Infantry of
the Phil. Army, operating as guerilla unit in Abra. Simultaneously upon his appointment, Beronilla
received a memorandum which authorized him to appoint a jury of 12 bolo men to try persons
accused of treason, espionage and aiding or abetting the enemy.
Upon the return of Borjal and his family to Abra, to escape bombing in Bangued, he was placed
under custody and tried and sentenced to death by the jury based on various complaints made
by the residents. Beronilla reported this to Col. Arnold who replied, saying I can only
compliment you for your impartial but independent way of handling the whole case.

Two years thereafter, Beronilla, along with the executioner, digger and jury, were indicted for the
murder of Borjal. Soon after, President Manuel Roxas issued Executive Proclamation 8, which
granted amnesty to persons who committed acts in furtherance of the resistance to the enemy
against persons aiding in the war efforts of the enemy.

The rest of defendants applied and were granted amnesty, but Beronilla and others were
convicted on the grounds that the crime was made on purely personal motives and that the
crime was committed after the expiration of time limit for amnesty proclamation.

Issue: W/N the defendant-appellants actions are covered by justifying circumstances for
obedience to lawful order of superior

20
Held:
Yes. The accused acted upon orders of their superior officers, which as military subordinates,
they could not question and obeyed in good faith without the being aware of its illegality.
The evidence is sufficient to sustain the claim of the defense that arrest, prosecution and trial of
Borjal was done in pursuant to express orders of superiors. Additionally, it could not be established
that Beronilla received the radiogram from Colonel Volckmann, overall area commander,
which called attention to the illegality of Borjals conviction and sentence. Had Beronilla known
the violation, he would not have dared to report it to Arnold. The conduct of the accused also
does not show malice on their part because of the conduct of the trial, defense through counsel
given to Borjal, suspension of trial based on doubts of illegality and death sentence review sent
to the superior officers.

Criminal intent then could not be established. The maxim here is actus non facit reum, nisi mens
rea (Crime is not committed if the mind of the person performing the act complained of to be
innocent).
Additionally, the lower court should not have denied their claim to the benefits of the Guerilla
Amnesty Proclamation No. 8 inspite of contradictory dates of liberation of La Paz, Abra. Even if
the dates were contradictory, the court should have found for the Beronila, et al because if there
are any reasonable doubt as to whether a given case falls within the (amnesty) proclamation
should be resolved in favor of the accused.

13.Criminal Law Case Digest: People vs PO3 Fallorina G.R. No. 137347 March 4, 2004
People vs PO3 Fallorina
G.R. No. 137347
March 4, 2004

Criminal Law

Criminal Case Digest

Facts:

At about 2:30 p.m. of September 26, 1998, Vincent Jorojoro, an eleven-year old minor and the third child
of Vicente and Felicisima Jorojoro, residing at Sitio Militar, Brgy. Bahay Toro, Project 8, Quezon City, asked
permission from his mother Felicisima if he could play outside. She agreed. Together with his playmate
Whilcon Buddha Rodriguez, Vincent played with his kite on top of the roof of an abandoned carinderia
beside the road.

Beside the carinderia was a basketball court, where a fourteen-year old witness Ricardo Salvo and his
three friends, were playing basketball. Ricardo heard the familiar sound of a motorcycle coming from the
main road across the basketball court. Cognizant to Ricardo of the appellant, PO3 Ferdinand Fallorina, a
Philippine National Police (PNP) officer, detailed in the Traffic Management Group (TMG), knew that he
abhorred kids playing on the roof, since one of his friends was previously been scolded by the appellant
before.

Ricardo called on Vincent and Whilcon to come down from the roof. When PO3 Fallorina saw them, the
former stopped his motorcycle, he shouted and badmouthed at them. After hearing the shouts of the
appellant, Whilcon rushed to jump off from the roof while Vincent was lying on his stomach on the roof

21
flying his kite. When he heard the appellants shouts, Vincent stood up and looked at the latter. As soon as
Vincent turned his back, ready to get down from the roof, suddenly, the appellant pointed the .45 caliber
pistol towards the direction of Vincent and fired a shot. Vincent fell from the roof, lying prostrate near the
canal beside the abandoned carinderia and the basketball court.

The appellant approached Vincent and carried the latters hapless body in a waiting tricycle and brought
him to the Quezon City General Hospital. Vincent was pronounced dead on arrival caused by a single
gunshot wound in the head.

Issues:

(a) Whether or not the appellant is exempt from criminal liability?

(b) Whether or not the appellant can offset the aggravating circumstance of taking advantage of public
position from a mitigating circumstance of his voluntary surrender?

Held:

The Office of the Solicitor General (OSG) cites that the basis for exemption from a criminal liability under
Article 12, paragraph 4 of the Revised Penal Code (RPC), is the complete absence of intent and
negligence on the part of the accused. For the accused to be guilty for a felony, it must be committed
either with criminal intent or with fault or negligence.

Thusly, the elements of exempting circumstances are (1) a person is performing a lawful act; (2) with due
care; (3) he causes an injury to another by mere accident; and (4) without any fault or intention of causing
it.

In the case at bar, the Court a quo erred in inequitably appreciating exculpatory and inculpatory facts
and circumstances which should have been considered in favor of the accused. The court also failed to
appreciate the mitigating circumstance of voluntary surrender in favor of the accused since it was only
after three days that the appellant gave himself up and surrendered his service firearm. And lastly, the
court considered the aggravating circumstance of taking advantage of his position by the accused.

On January 19, 1999, the trial court rendered judgment convicting the appellant-accused of murder,
qualified by treachery and aggravated by abuse of public position. The trial court did not appreciate in
favor of the appellant the mitigating circumstances of voluntary surrender.

The Regional Trial Court of Quezon City, Branch 95, found the accused PO3 Ferdinand Fallorina y Fernando
GUILTY beyond reasonable doubt of the crime of murder defined and penalized by Article 248 of the RPC,
as amended by the Republic Act No. 7659, and in view of the presence of the aggravating circumstance
of taking advantage by the accused of his public position (par. 1, Art. 14, RPC). Hence, the accused is
hereby ordered to indemnify the heirs of late Vincent Jorojoro, Jr. the amounts of actual damages of
P49,174.00 (paid for funeral services); P50,000.00 for moral damages; P25,000.00 as exemplary damages;
and P50,000.00 as death indemnity. The court a quo sentenced the appellant to suffer the Death Penalty.

14. 159 Phil. 212

FERNANDEZ, J.:

22
This is an automatic appeal from a decision of the Circuit Criminal Court, Seventh Judicial District in Criminal
Case No. CCC-VII-922 Rizal, dated October 11, 1971, the dispositive part of which reads as follows:

"WHEREFORE, finding the accused Benjamin Ong y Kho and Bienvenido Quintos y Sumaljag, GUILTY,
beyond reasonable doubt of the crime of Kidnapping with Murder as defined under Article 248 of the
Revised Penal Code, in relation to Article 267 thereof, as charged in the Information, the Court hereby
sentences each one of them to suffer the penalty of DEATH; to indemnity the heirs of the deceased Henry
Chua, the amount of P12,000.00; to pay moral damages in the amount of P50,000.00, and another
P50,000.00 as exemplary damages jointly and severally; and to pay their proportionate share of the
costs."[1]
The information filed by the Provincial Fiscal of Rizal, B. Jose Castillo against (1) Benjamin Ong y Kho, (2)
Bienvenido Quintos y Sumaljag, (3) Fernando Tan, alias, "Oscar Tan," and (4) Baldomero
Ambrosio alias "Val", the latter two being then at large, reads:

That on or about April 23 to April 24, 1971, inclusive, in the municipality of Paraaque, province of Rizal,
Philippines, and within the jurisdiction of this Honorable Court, the above-named accused, being then
private individuals, conspiring and confederating together and mutually helping one another did then
and there wilfully, unlawfully and with treachery and known premeditation and for the purpose of killing
one Henry Chua and thereafter extorting money from his family through the use of a ransom note,
kidnap(ped) and carry(ied) away said Henry Chua, initially by means of a friendly gesture and later
through the use of force, in an automobile, and later after having taken him to an uninhabited place in
Caloocan City, with the use of force detained him (Henry Chua) and killed) him in the following manner
to wit: The accused after gagging and tying up Henry Chua and repeatedly threatening him with death,
assured him that if he would write and sign a ransom note for the payment by his family of the sum of
$50,000.00 (US), he would not be killed and would be released upon receipt of the ransom money, but
after said Henry Chua agreed and did execute such a ransom note, he was again gagged and tied up
by the accused, and thereafter stabbed in the abdominal region, several times with an icepick, inflicting
upon him (Henry Chua) mortal wounds on his vital organs, which directly caused his death.

"All contrary to law with the following generic aggravating circumstances:

(a) Evident premeditation;


(b) Grave abuse of confidence;
(c) Nighttime;
(d) Use of a motor vehicle;
(e) Use of superior strength; and
(f) Cruelty."[2]

Personal Circumstances of the Two Appellants

At the time of the trial before the lower court in September of 1971, the accused Benjamin Ong was 31
years old, employed with the Acme Shoes, Rubber and Plastic Corporation, a firm owned by his brother-
in-law, Chua Pak, for the past 11 years, the last 6 of which was as an assistant manager. He was already
receiving a monthly salary of P1,800.00 excluding yearly bonuses of P30,000.00 and other representation
allowances or a total annual income of from P60,000.00 to P70,000.00. He had his elementary schooling
at the Assumption Academy in San Fernando, Pampanga; his first and second years of high school at
Chiang Kai-shek High School in Manila; and his third and fourth years at the Mapua Institute of
Technology. He was a third year Commerce student, majoring in accounting at the University of the East,
when he quit schooling in 1959. He married Athena Caw Siu Tee Ong on November 25, 1962 at the St.
Jude Catholic Church, by whom he already had four children: Connie Louis, 7 years old; Dennis, 5 years
old; Edgar, 3 years old; and Fanny, 1 year old.[3]

23
On the other hand, accused Bienvenido Quintos was 39 years old, single, an unlicensed surveyor and
computer for two years already at the Robes Francisco Realty Corporation with a relatively "small"
income. He was a third year engineering student when he stopped studying. In 1954 he was charged of
Resisting Arrest and Assault Upon an Agent in Authority but this case was settled amicably. [4]

Brief Synopsis of the Testimony of the Prosecution's Witnesses

The prosecution presented several witnesses to prove its charge of kidnapping with murder. First to testify
was Patrolman Marciano Roque of the Crime against Property Division of the Detective Bureau of the
Caloocan City Police Department who declared that: He knew Benjamin Ong for about 6 years already
because he usually investigated theft and robbery cases at the Acme Firm and at times received some
money from Ong. In a series of 6 meetings with Benjamin Ong starting from the first week of April, 1971,
Benjamin Ong confided to him his plan to get a man who cheated him in gambling by as much as
P150,000; that he would ask for money from the latter's parents; and that after which, he would kill the
victim. Benjamin Ong's determination was shown when his godson was even introduced to him as one
who would help him. Benjamin Ong brought him to Barrio Makatipo in Novaliches, Caloocan City and
described it as a suitable place where to bring the victim. Ong also told him that he had acquired a bag,
flashlight and a piece of cloth. He was prevailed upon by Banjamin Ong to participate in his plan assuring
that he could resign from the government service once the money is collected. Patrolman Roque
revealed this plan to his Division Chief, Capt. Duenas, the Officer-in-Charge, Lt. Manapat, and the Chief
of Police, Celestino Rosca. However, the three did not believe that Benjamin Ong had the guts to do
it. After the incident, Patrolman Roque said that he and Police Chief Rosca met with Atty. Nestor Gonzales
of the National Bureau of Investigation to supply the early leads in this case although they did not find a
trace of the crime when they went to Barrio Makatipo.[5]

Miss Ligaya Tamayo testified next. She declared that: She worked as an entertainer at the Wigwam
Nightclub in Paraaque, Rizal and knew Henry Chua very well. At around 1:30 o'clock in the early morning
of April 24, 1971, she and Miss Mickie Yaro had Henry Chua and Benjamin Ong for their guests. The two
talked in Chinese and had some drinks. Benjamin Ong showed her a check in favor of Henry Chua which
he claimed that the latter won in a gambling game. She, however, did not actually see him give it. At
around 1:30 that same morning, she accompanied the two to the door and saw them leave the place
and ride in a Mustang car.[6]

Sy Yap, older brother of Henry Chua, was the third witness. He testified that: He was with Atty. Nestor
Gonzales and other agents of the NBI on September 2, 1971 in Barrio Makatipo after Benjamin Ong
pinpointed the place of burial, and there he saw the decomposing body of the victim under the ground,
immersed in water. He saw and identified the following personal effects found with the body: a white
gold watch which stopped at the hour of 6:22 and date of "24"; Driver's License No. 32219 with the name
of Sy Sing Biok alias Henry Chua; Diner's card Diner Group 0004149-1; pass issued by the Bureau of Customs
for Henry Chua dated January 19, 1971; receipt for payment of the license of the car; residence certificate;
lighter; wallet; currencies in different denominations; shirt jacket; pair of shoes; socks; brief; undershirt; T-
shirt; and trousers with a mark "Especially tailored for Henry Chua, 2-2-71, No. 95812."[7]

Dr. Ricardo G. Ibarrola, Jr., Medico-Legal Officer of the NBI, appeared as the fourth witness. He testified
on his post mortem examination made on September 2, 1971 at La Funeraria Paz, of the deceased Henry
Chua, 31 years old, single, and on his necropsy report, Exhibit "M". He said that the deceased sustained
two wounds on the liver and large intestine caused by a long pointed cylindrical instrument similar to an
icepick. He added that most likely, the assailant was in front of and on a higher level than the
victim. Although this did not appear in his report, he theorized that the two wounds were not the

24
immediate cause of death since there was only a slight degree of hemorrhage in the vicinity of the
punctured wounds. He said that the liver and large intestine had no sufficient time to bleed because
something else must have happened which was the asphyxiation or suffocation of the victim due to his
burial.[8] He stated, however, in his necropsy report, Exhibit "M", that the cause of death of the deceased
was "punctured wounds of the abdomen."

Miss Clarita Teh, travel agent of Skyways Travel Service located at Ongpin St., Sta. Cruz, Manila, declared
that: At about 4:00 p.m. of April 22, 1971, Benjamin Ong called her up by phone to ask for a reservation
ticket for Hongkong and Taipei. On the morning of April 23, 1971, Benjamin Ong went to her office but
forgot to bring along his papers including his Alien Certificate of Registration. In the afternoon of April 24,
1971, Benjamin Ong went back to the office, this time with the pertinent papers plus P4,000 cash. She said
that he changed his destination from that of Hongkong and Taipei to that of Canada. However, he
needed P7,000 for this purpose. On April 29, 1971, Mrs. Ong got back the P4,000 because the latter said
that her husband did not have enough money.[9]

Patrolman Gener S. Estrella, municipal policeman of Baliuag, Bulacan, followed next on the witness
stand. He stated that on April 25, 1971, he was on his tour of duty from 4:00 o'clock to 8:00 o'clock a.m. at
the poblacion when he received information that an unidentified car was parked in a gasoline station. He
therefore sought the company of Patrolman Ceferino Castro and they went to Barrio Tibag where they
saw the locked Mustang car parked in a gasoline station with plate number 16-02B, L-P.C., series '71. They
reported the matter to their head, Lt. Herminio Angeles. [10]

Severo "Boy" Roslin, mechanic, gave the next testimony. He knew Fernando Tan since 1965. On April 29,
1971, early morning, he saw Fernando Tan and another, introduced to him as Alfredo Hernandez, who
happened to be Banjamin Ong. Fernando Tan requested him to bring them to the airport and obtain
airplane seats for the Visayas. He accompanied them but they failed in this endeavor so that they
proceeded to the pier. Likewise, they were frustrated in getting a passage to the South. They ended up
taking a train ride to Lucena City. Roslin said that he went back to Manila that same day. On May 1, 1971,
he and Fernando Tan went to the house of Bienvenido Quintos near Abad Santos St. in Manila. They did
not see him so that they had to come back at noon. They then took him with them and, after passing by
a laundry shop, they went to Singalong where they picked up Benjamin Ong at around 7:00 p.m. Roslin
claimed that they were using his Chevy car. They went to Barrio Balugo, Oas, Albay and stayed at his
parent's house. He, Quintos, and Tan stayed there for one half month where they took themselves into
swimming at the river. They left Benjamin Ong there.[11]

Enrique Lacanilao, an NBI agent, testified that: Exhibits "N" and "O" are the voluntary written statements
signed respectively by Benjamin Ong on September 1, 1971 and by Bienvenido Quintos on September 3,
1971. He said that Benjamin Ong pinpointed to them the place of burial at Barrio Makatipo, and Sy Yap
was with them during the examination. They found the mouth of the victim gagged and his hands tied. It
was in a state of decomposition. The victim's body was facing downward with the buttocks protruding
up. The hands were tied just above the chest while the feet were far apart. The buttocks were one foot
from the surface while the face was one and a half feet below facing down. There were no houses in the
area which he believed was the Araneta subdivision. He directed the reenactment of the crime. It
appeared in their reenactment that Fernando Tan and Bienvenido Quintos were the ones who grabbed
Henry Chua from his Mustang car when Benjamin Ong was urinating; that the victim's mouth was gagged
while his hands were tied at the back; that during the making of the ransom note, Tan was holding the gun
while Quintos was focusing the flashlight; that afterwards, Henry Chua's hands were tied again, this time in
front; that he was stabbed after he was made to lie down facing up; that Baldomero Ambrosio and
Bienvenido Quintos pulled the victim to the hole; that Baldomero Ambrosio shovelled while Bienvenido
Quintos held the flashlight; that at the time the ransom note was being prepared Benjamin Ong was near
the car, about 50 meters from the hole, so that his person did not appear in the picture of the reenactment

25
of this portion. Benjamin Ong was taken by the NBI into custody from the 2nd PC Zone on September 1,
1971 at around 6:30 in the evening whereupon at 10:00 p.m. of that same night, his written testimony was
taken down up to past 12:00 midnight. He had a small bandage around his wrist because of an attempted
suicide on his part. Bienvenido Quintos, on the other hand, he said, was arrested on September 3, 1971
and his extrajudicial statement was taken on the same day at around 7:00 or 8:00 p.m. [12]

Diego H. Gutierrez, also an NBI agent, testified last for the prosecution. He identified Exhibits "Q" and "R" as
the voluntary supplementary extrajudicial statements respectively of Bienvenido Quintos and Benjamin
Ong. Gutierrez' testimony focused on Bienvenido Quintos' admission that the hole was dug and covered
with fresh twigs after the group's second meeting at the Barrio Fiesta Restaurant.[13]

Brief Synopsis of the Testimony of the Witnesses for the Defense

The defense started the presentation of their evidence with the testimony of Dr. Mariano P. Lara, retired
Chief Medico-Legal Officer of the Manila Police Department. His testimony centered on the matter of
asphyxiation. He said that asphyxiation as the possible cause of death was nowhere reflected on the
necropsy report of Dr. Ibarrola of the NBI; and that the death of the victim could have been due to shock
as a result of the wounds inflicted on him.[14]

Rene Aguas, BIR examiner and first cousin of Bienvenido Quintos, then testified. He said that he went to
the NBI on September 8, 1971 in order to follow up the clearance papers of his deceased father. By
coincidence, he discovered that Quintos was detained there, so, he tried to get in touch with him. He
gathered that Quintos was "okay" although later on the latter revealed that he was hurt also.

Artemio R. Quintos an engineer and father of accused Bienvenido Quintos, followed next. He said that he
visited his son on September 3, 1971 along with Atty. Bonicilla at around 7:00 p.m. at the NBI. The guard
refused to tell him where his son was so that the following day, September 4, he went back to the NBI in
the morning as well as in the evening. Still he did not find his son. On September 5, he delivered clothes
for the use of his son to the jailer, Benjamin Laforteza, and was issued a receipt therefor. On September 6,
he brought a letter addressed to the Director of the NBI requesting him that he be allowed to see his son. It
was only on September 7, at 4:00 p.m. he claimed, that he met his son. He said that Bienvenido Quintos
showed to him his stomach with some bluish discoloration at the navel. On that day, he also received his
son's dirty clothes and found bloodstains on it.[15]

Bienvenido Quintos then took the witness stand. He revealed that he came to know Fernando Tan when
they were still in Dagupan City long time ago. He said that he was invited on April 23, 1971 by Fernando
Tan and that they met at around 7:00 p.m. of that day. They proceeded to the Barrio Fiesta Restaurant in
Caloocan City where he was introduced to Benjamin Ong and Baldomero Ambrosio for the first time. At
9:00 p.m., they went to Brown Derby Supper Club in Quezon City after which they proceeded to Amihan
Nightclub at around 10:30 p.m. at Roxas Boulevard. He, Fernando Tan, and Baldomero Ambrosio were left
in the car. Later, Benjamin Ong went out of the Amihan Nightclub and took Fernando Tan with
him. Fernando Tan returned and after a while he was invited to the nearby Wigwam Nightclub. They
hurriedly left the place and Fernando Tan took the front seat of the Biscayne car while he took the back
seat and followed a certain car. When that car stopped, he saw Benjamin Ong vomitting. Fernando Tan
and Baldomero Ambrosio went down and Fernando Tan pulled out his gun. The victim was dragged and
forced into the rear part of their car. The victim's hands and feet were tied by Baldomero Ambrosio while
the mouth was gagged by Fernando Tan with a flannel cloth. Bienvenido Quintos made clear in his
testimony that the victim was lying on his back inside the car so that his face was up and his hands were
on his breast. Fernando Tan then threatened him with his gun should he not cooperate with them. At
Barrio Makatipo, the victim laid down on the ground and Benjamin Ong got the shovel and flashlight and

26
gave them to Fernando Tan. The victim was made to walk a little distance and then lie down again face
up. Benjamin Ong gave to Fernando Tan an icepick who then gave it to Baldomero Ambrosio and in turn
gave it to him. He refused to stab the Victim so that he returned it to Fernando Tan who made the actual
stabbing on the victim's chest twice: According to him, there was already a hole in that place. He also
claimed that Exhibit "O" was not a voluntary statement of his and that he was maltreated by more or less
5 men. He said that he went to Oas, Albay on May 1, 1971 but that he was never contacted by the group
between April 24; and 30. At a certain point during the proceedings, the court suspended his testimony
for about 15 minutes after he complained of an aching head.[16]

Benjamin Ong testified last for the defense. He related that Henry Chua was a friend and that they were
slightly related to each other. He felt that he was cheated because he was the only one who continuously
lost in their mahjong sessions. Henry Chua's group, including Ko King Pin, Go Bon Kin and Marcelo Tanlimco,
went to his office and humiliated him there. On April 21, 1971, Henry Chua called him up by phone and
invited him to the Amihan Nightclub where he could settle the gambling debt. He admitted responsibility
for Henry Chua's death but emphasized that his purpose was merely to kill him. He added that nothing
was taken from the body of the victim. He asked the assistance of Fernando Tan and Baldomero Ambrosio
who merely drove the car. He denied the testimony of Patrolman Marciano Roque regarding his revelation
of his plan. He believed that Henry Chua knew that he had a grudge against him during that fatal day. He
waited for them to dig and cover the hole which took about one hour and a half after the stabbing. He
attempted suicide by slashing his wrist 7 or 8 times while he was still in the custody of the P.C. at Camp
Vicente Lim in Laguna. He was also brought by the NBI to the Salem Motel where he was investigated
from 8:30 in the evening up to 5:30 in the morning of the next day. Exhibit "N", his extrajudicial statement,
was taken while he was groggy and very weak. He likewise pinpointed the grave. At a certain juncture
during Benjamin Ong's testimony, his counsel sought the court's permission to exclude the public from the
hearing because Ong's wife would testify on something that would constitute a "great shame" to their
family. Benjamin Ong, however, refused to go ahead with said testimony. Benjamin Ong further claimed
that he decided to kill Henry Chua on April 22, 1971. He was hurt by the threatening words on the part of
the victim which humiliated him and, as such, he was forced to resign from his job. He went to the Skyways
Travel Service only after the incident. He, however, changed his destination and wanted to go instead to
Canada and Europe. The reason why he was not able to pursue his departure was because Sy Yap called
him up and asked him about his brother's whereabouts so that he seriously felt that the authorities were
already after him. He left Manila on April 29, 1971 and went to Legaspi City with Fernando Tan but found
no acquaintance there so that they went back to Manila. It was Fernando Tan who contacted Boy Roslin
and Bienvenido Quintos after which they went to Oas, Albay and stayed there for about two to three
days. He hid himself on top of the mountain with an old man. Furthermore, he said that Henry Chua was
aware that he resented him. Benjamin Ong likewise denied having called Fernando Tan at anytime, to
come in with him to the nightclub.[17]

Non-Conflicting Facts

Non-conflicting facts, as shown in the testimonies of the accused and witnesses in open court, and
reiterated in the respective briefs of the parties, are as follows: For more or less one year and a half prior
to the dreadful incident, the accused Benjamin Ong used to play mahjong with the deceased Henry Chua
and the latter's companions, Ko King Pin, Go Bon Kim and Marcelo Tanlimco. In those sessions, he lost
substantially that at one time, it amounted to as much as P150,000.00 He suspected that he lost in unfair
games and was completely cheated by Henry Chua and the latter's companions, who made things worse
by pressing him to pay his gambling debt with a threat of bodily harm upon his person and that of his
family. The deceased and his companions embarrassed Benjamin Ong, incident after incident, especially
when they went time and again to Benjamin Ong's office at the Acme Shoes, Rubber and Plastic

27
Corporation to confront him. The extent of his embarrassment was made manifest by the fact that he had
to resign from his job.

On April 21, 1971, Henry Chua repeated his demands for early settlement of his gambling debt and, as
such, invited Benjamin Ong to see him on April 23, 1971 at the Amihan Nightclub and bring with him the
money owed (P150,000.00). That same day that Henry Chua phoned Benjamin Ong, the latter contacted
and sought the assistance of Fernando Tan, a technical supervisor also of the Acme Firm. Benjamin Ong
told Fernando Tan about his grudge and plans against Henry Chua in order to avenge the embarrassment
and humiliation he suffered before the eyes of his subordinates.

Fernando Tan, who incidentally, owed Benjamin Ong his job,[18] was very accommodating and he shared
Ong's feelings against Henry Chua. And, according to Benjamin Ong, Tan said "Why not just kill him."[19] Tan
immediately contacted Baldomero Ambrosio, Benjamin Ong's godson in marriage and a former Acme
employee, and likewise called upon his boyhood friend Bienvenido Quintos at the latter's office at the
Robes Francisco Realty Corporation.

On April 23, 1971, the four met at the Barrio Fiesta Restaurant in Caloocan City and finalized their plan to
liquidate Henry Chua. The group, riding in Benjamin Ong's Biscayne car; then went to the Amihan
Nightclub and arrived there at past nine o'clock in the evening. The two, Benjamin Ong and Henry Chua
met there and had a couple of drinks. Benjamin Ong asked for patience and leniency with regard to his
indebtedness and ample time for its settlement.

From the Amihan the two went to the nearby Wigwam Nightclub where they tabled two hostesses, Ligaya
Tamayo and Mickie Yaro and had some more drinks. At around 1:30 a.m. of the following day, April 24,
1971, the duo left the place and rode in Henry's Mustang car. Fernando Tan, Bienvenido Quintos and
Baldomero Ambrosio, riding in Ong's Biscayne car, followed the couple down Roxas Boulevard, then to
Quiapo and Quezon Boulevard Extension in Quezon City where, after passing the Sto Domingo Church,
they made a turn towards a dirt road leading to Del Monte Avenue. When they reached a dark and
secluded place, Benjamin Ong urged Chua to stop the car in order to urinate, to which the latter
obliged. It was at this time that the Biscayne car arrived and stopped in front of the Mustang car
whereupon Fernando Tan and Baldomero Ambrosio alighted with a flashlight and pretended to be
policemen. Fernando Tan poked his gun at Henry Chua and pulled him down from his Mustang car with
Baldomero Ambrosio giving him help. They then guided and forced him inside the rear part of the
Biscayne. He was made to lie, face up. His hands were tied and his mouth gagged with a flannel
cloth. Fernando Tan and Bienvenido Quintos then rested their feet on him. Baldomero Ambrosio drove
the Biscayne while Benjamin Ong drove the Mustang and followed them from behind.

The group took Del Monte Avenue, Roosevelt Avenue, and then E. de los Santos Avenue, right to the North
Diversion Road, and right again to Novaliches until they reached a deserted place that looked like an idle
subdivision in Barrio Makatipo, Novaliches, Caloocan City. It was here that Henry Chua was stabbed twice
with an icepick, allegedly by Fernando Tan, and buried there with all his belongings with him consisting of
a Piagot watch, lighter, wallet containing P50 bills, driver's license, diner's card, etc.

After this, the group proceeded to Barrio Tibag, Balivag, Bulacan with Benjamin Ong and Fernando Tan
on the Mustang. There they left it locked near a gasoline station. The foursome then regrouped in the
Biscayne and proceeded back to Caloocan City where they separated at about 7:00 o'clock in the
morning.

On August 29, 1971, somewhere in Barrio Balugo, Oas, Albay, Benjamin Ong was arrested by operatives of
the 2nd PC Zone and later turned over to the NBI. On the other hand, Bienvenido Quintos was
apprehended on September 2, 1971 in his residence at Tayabas St., in Sta. Cruz, Manila by members of the

28
MPD and later turned over to the NBI also.

Important Points of Conflict

The prosecution adds more to what the defense claims and conflicts appear in various instances. One
such instance was the testimony of the first prosecution witness, Patrolman Marciano Roque of Caloocan
City, to the effect that one month or so before the execution of the crime, Benjamin Ong solicited his help
in consummating his plan. Patrolman Roque testified that he tried his best to convince Benjamin Ong to
desist but to no avail. It was this witness who revealed Benjamin Ong's plan to ask for money from the rich
family of the deceased and, with said money, he, Roque, could already resign from his job should he
participate.[20]

In his testimony before the lower court, Benjamin Ong vehemently denied having revealed such plan to
the witness.[21] However, in his brief, accused Benjamin Ong claims that this testimony if ever there was
such, does not reveal his intention to kill Henry Chua that early. At most, he said, it was a mere "infantile
thought of wishing someone dead" and no more.[22]

On this point, counsel for the accused Ong, argued as follows in their well-written brief:

"Pat. Roque has not categorically asserted that he was a friend of Benjamin Ong. They came to know
each other when he, as a policeman, investigated theft and robbery cases on the complaint of the Acme
Shoe and Rubber Corporation where Benjamin Ong worked as Assistant Manager. (pp. 5-7, t.s.n., Sept. 16,
1971) As to why Benjamin would reveal a plan to kidnap another to a policeman, in the absence of a
close and long association, is just too incredible to merit belief. Pat. Roque said that Benjamin Ong
"confided to me that I am the only person whom he can trust so he further enumerated a detail that he
intended to get a money and ask for the money from the parents of the victim." (Id., p. 10) As to why he
merited the trust of Benjamin Ong, he did not say.

"Pat Marciano Roque said that he has no criminal record (Id., p. 42). He has not conveyed to Benjamin
Ong any information that he is a gun for hire (p. 43), nor does he have that reputation (Id., p. 43). If he
were a criminal or he had a reputation as a professional killer, it is perhaps possible for one in Benjamin
Ong's position to have made the proposition to him. Moreover, when he was cross-examined on the
alleged intention to collect ransom, he committed material contradictions such as to raise serious doubt
on the veracity of his testimony. He could not categorically assert whether the alleged intention of
Benjamin Ong was to kill the victim first and demand money from his parents after, or detain him first, and
after receiving ransom money, kill the victim.

ATTY. QUISUMBING:

Your testimony is as follows: that he told you that after demanding the money to kill the man,
Q
you remember that?

A That was what he said.

In other words, this was not the way he told you, that he would grab the man so that he could
Q
get the money by extortion or by ransom?

29
He said that after having in his possession his intended victim he would demand some money
A
from his parents.

I will recall in your direct testimony ... you said that afterwards if he could get the money he will
Q
kill the man, that was your first testimony, which is correct?

A He lost one hundred fifty thousand.

And he needed money and so he would demand money from the father or parents of the
Q
victim, is that your testimony?

A Yes, sir.

Q And afterwards he wanted to kill the man?

A No, sir.

Q And so what is your testimony now?

After he got the man he will demand money from the parents or ransom money from the
A
parents of the victim.

Q So it is the other way. He first would kill the man and afterwards get the money.

ATTY. DE SANTOS:

The question is misleading.

COURT:

Answer.

WITNESS:

A No, sir, he said that after receiving the money the man may be killed.

Q Is that your testimony? That he will kill the victim or the victim may be killed?

A No, sir.

Q So which is which?

A He will kill the victim?

30
A After getting the money?

A Yes, sir. (pp. 38-41, t.s.n., Sept. 16, 1971)"


Another point of conflict is the claim of the prosecution that a ransom note was indeed written and copied
by Henry Chua from a prepared note before the latter was icepicked and buried. It appears that co-
accused Bienvenido Quintos stated in his supplementary extrajudicial statement before the NBI that:

"Yes sir. After we have brought victim some meters away from the road, FERNANDO TAN ordered victim
to lie face down on the ground at the same (time) he untied victim and removed the gag while his gun
was still pointed at the head of Victim. Thereafter he ordered the victim to copy a prepared ransom note
in a piece of yellow paper saw the figure $50,000.00 because I was holding then the flashlight. It was only
after the ransom note was written and was submitted to BENJAMIN ONG that FERNANDO TAN returned to
us."[23]
This is hearsay as against Benjamin Ong and Ong vehemently denied the same in his testimony, in open
court when he said upon questioning:

In this statement Exhibit "N", you admitted that Henry Chua was taken from the Mustang car
and transferred to the Viscain (sic) car and then brought to that uninhabited place in Barrio
"Q
Makatipo; what was your purpose in having the late Henry Chua taken from his car and
brought to Makatipo?

A My purpose was just to kill him, and there is (sic) not going to be any delay.

"Q Was there any purpose of detaining him for sometime?

xxx xxx xxx

A No, there was no purpose to detain him any further."[24]

Also, in his extrajudicial statement, he said:

When you hatched the plan to kill HENRY CHUA, did it ever occur to you to demand or ask for
"Q
any ransom money from the family of HENRY CHUA?

A Never, the question of ransom money never entered my mind."[25]


Admittedly, no such genuine ransom note was received by the family of the deceased. Undoubtedly, its
presence in the crime could aggravate it, allowing the imposition of the capital punishment of death. [26]

Also conflicting is the matter of Bienvenido Quintos' participation at the time Henry Chua was dragged
into the Biscayne car. The briefs of both parties tend to show that it was Fernando Tan and Baldomero
Ambrosio who pulled Henry Chua out off his Mustang car, forced him into the Biscayne car, tied and
gagged him.[27] However, Agent Lacanilao testified that in the reenactment of the crime it was shown that
Bienvenido Quintos and Fernando Tan were the ones who dragged Henry Chua out of his car. [28] Added
to this is the claim of Benjamin Ong that Baldomero Ambrosio merely drove the Biscayne for the group. [29]

31
The prosecution likewise claims in its brief that as early as a week before the incident, the group already
chose a site and prepared a hole where to bury Henry Chua;[30] that this group was in constant search of
the victim along the nightclub row in Roxas Boulevard during the succeeeding evenings but failed to see
him;[31] that a day before the unfortunate evening, Ong contacted Miss Clarita Teh of the Skyways Travel
Service at Ongpin St., Sta. Cruz, Manila, and asked for a booking for Hongkong and Taipei, and deposited
P4,000.00 therein.[32] Similarly, it is alleged that on April 29, 1971, a few days after the incident, Tan and Ong
contacted Severo "Boy" Roslin, a long-time friend of Tan, to help them obtain airplane seats for the Visayas,
but they failed;[33] that they also proceeded to the pier to seek passage to the South on a boat but they
were likewise frustrated;[34] that instead, they took a train ride to Lucena City where Roslin left them and
after which, they continued to Legaspi City;[35] that finding no acquaintance there, they went back to
Manila;[36] that on May 1, 1971, Tan again engaged Roslin's services and with the latter driving his car, they
picked up Quintos and Ong and went to Barrio Balugo, Oas, Albay and stayed there in the house of Roslin's
parents;[37] that Ong was left there while Roslin, Tan and Quintos went back to Manila.[38]

A reenactment of the crime was had by Benjamin Ong, Bienvenido Quintos and some NBI and MPD agents
who played the role of their co-accused Fernando Tan and Baldomero Ambrosio.[39]

The trial of this case in the lower court proceeded with commendable speed, although separate trials for
the two accused who had been arrested so far at that time were held upon the latter's request. Both
entered a plea of "not guilty" to the crime charged upon arraignment on September 4, 1971. However, in
the case of Benjamin Ong, he invoked the doctrine laid down in the case of People vs. Yturriaga[40] to the
extent that the prosecution should not nullify the mitigating circumstance of a plea of guilty, by
counteracting it with "unfounded allegations" of aggravating circumstances in the information. In other
words, he admitted his guilt in so far as the crime of simple murder was concerned. [41]

Before this Court, the accused Benjamin Ong maintains that:

"The Court a quo erred in finding the accused guilty of the crime of kidnapping with murder because

(a) There was no evidence offered against the accused which would prove that the crime of kidnapping
was committed at all;

(b) Kidnapping cannot be complexed with murder.

(c) In those cases where the Supreme Court convicted the accused of Kidnapping with Murder, there was
shown an intention to deprive the victim of his liberty, and it was held that the kidnapping was a necessary
means to commit the crime of murder.

II

"The court a quo erred in finding that the killing of the deceased was attended by the generic aggravating
circumstances of

(a) Abuse of superior strength;


(b) Nighttime;
(c) Uninhabited place;
(d) Abuse of confidence;

32
(e) Use of motor vehicle; and
(f) Cruelty.
and the qualifying circumstances of

(a) Alevosia
(b) Evident premeditation.

III

"Assuming that the killing of Henry Chua was attended by the aggravating circumstance of alevosia, the
aggravating circumstance of abuse of superior strength and nighttime, if present, are absorbed by
treachery.

IV

"The court a quo erred in not appreciating (a) plea of guilty, and (b) circumstances of a similar nature or
analogous to Article 13, paragraphs 1 to 9 of the Revised Penal Code as mitigating.

"The court a quo erred in imposing the death penalty upon the accused.

VI

"The court a quo erred in sentencing the accused to pay excessive damages."[43]

For his part, the accused Bienvenido Quintos argues that:

"1. The lower court erred in giving full weight and credit to the extrajudicial statement of the defendant-
appellant.

"2. The lower court erred in not finding that there was no conspiracy between defendant-appellant
Bienvenido Quintos and the other accused.

"3. The lower court erred in not acquitting defendant-appellant Bienvenido Quintos."[44]

OUR RULING

The Evidence on the Alleged Writing of a Ransom Note is Insufficient to Support a Finding in Favor of the
Prosecution:

First, Benjamin Ong vehemently denied asking for ransom.

33
"In the extrajudicial statement of Benjamin Ong, he was asked this question: 'Q. When you hatched the
plan to kill HENRY CHU A, did it ever occur to you to demand or ask for any ransom money from the family
of HENRY CHUA?' to which he answered: 'Never, the question of ransom money never entered my mind.'"
(Question No. 5, Exh. N.)
Secondly, no ransom note was presented as evidence by the prosecution, nor did the latter show that a
demand for money was made upon the family of the victim. In the case of People vs. Manzanero,
Jr.,[44] We held:

"Furthermore, what could have been the motive for the kidnapping? According to the trial court, the
ransom money was needed by Manzanero to defray the huge expenses for the day-to-day living of his
lawful wife and seven children, and of his mistress and his five children by her, and his repair shop that was
earning only about P1,000 monthly could hardly meet the salaries of his 16 workers and mechanics. But is
it credible that Manzanero, 'being the intelligent and shrewd man that he appears to be,' according to
the trial court, could even have entertained the illusion that the kidnapping that he was to perpetrate so
clumsily and amateurishly would be profitable to him, and he could escape from criminal
prosecution? And what is strange is, if the ransom note was indeed written, why was it never presented in
evidence? The claim that it was lost is unbelievable. That ransom note, if it ever existed, was the most
important piece of evidence that could support the prosecution's theory that the kidnapping was for
ransom. Certainly, that piece of evidence should be kept and preserved. No plausible explanation was
given how that ransom note got lost. Neither the father nor mother of Floresita was made to testify
regarding the alleged ransom note.

"Moreover, if ransom was the purpose of the kidnapping, why did Manzanero so easily, and without
apparent reason, give up his alleged criminal enterprise, when he could have pursued it to a successful
end? If there was really that ransom note, and that ransom note was sent, the most logical thing that
Manzanero would have done was to send instructions to Floresita's family on how, when, and to whom the
ransom money should be delivered. There is no evidence that Manzanero ever made any follow up in
order to get the ransom.

"Furthermore, barely two days after the alleged kidnapping for ransom, Manzanero, without having
obtained even part of the ransom money, released Floresita. Would a kidnapper, as Menzanero was
alleged to be, readily release the victim without realizing his purpose?" (Italics Supplied)
Thirdly, the extrajudicial statement of accused Quintos wherein he stated that Fernando Tan ordered Henry
Chua to prepare a ransom note wherein he saw the figure S50,000.00, is tainted with serious doubts due to
the apparent maltreatment that Quintos received from the NBI and MPD men on September 3, 1971.[45] The
medical certificates and case record[46] issued by the Philippine General Hospital support the findings and
remark of the examining physician, Dr. Florencio Lucero, that in the person of accused Quintos,
"intramascular hematoma is evident." BeSides, it is hearsay and therefore incompetent evidence against
Benjamin Ong. And in the reenactment, as testified to by NBI agent Lacanilao, while the ransom note was
being prepared, Benjamin Ong was about 50 meters away from the place where the note was being
prepared.

Fourthly, although both parties in their briefs agree that the victim's hands ere tied after he was shoved into
the rear floor of the Biscayne car, neither, makes a categorical claim that the hands were tied at his
back. In fact Acting Solicitor General Hector C. Fule submits in his brief that the victim was made to lie
down "face up".[47] This leads to the conclusion that the rope around the victim's hands was never removed
at any instance up to the time that he was buried and exhumed. This discounts the idea that before the
victim was made to a copy prepared ransom note the hands at his back were tied, and after the writing,
his hands were again tied, this time in front. Bienvenido Quintos in open court positively stated that the
victim was made to lie on his back inside the car and hands tied on his breast. [48] The contrary evidence
on this point are those of Agent Lacanilao on the reenactment of the crime which was based on the

34
extrajudicial statement of Bienvenido, Quintos.[49] However, as shown above, this statement is of dubious
veracity.

Finally, that appellants never intended to make money out of the murder of Henry Chua, can be clearly
deduced from the fact that Chua was buried with everything in his person; and during the exhumation of
his body, his brother, Sy Yap Chua, identified the articles found in the body of the deceased, such as a
Piaget watch worth around P10,000.00 (Exh. B), a wallet together with money, with P50 bills and other
denominations.

In the light of the foreging facts and circumstances, We cannot give any credence to the testimony of
Patrolman Roque that about the first week of April, 1971, Benjamin Ong confided to him his plan to get a
man who cheated him in gambling by as much as P150,000.00; that he would ask for money from the
latter's parents and after which he would kill the victim. And the facts brought out on cross examination
of this witness, which We have discussed earlier, show the incredibility of Ong confiding to Patrolman Roque
his criminal intention, particularly, his intention to ask money from the parents of the intended victim. As a
matter of fact, this witness, on cross examination, got lost, so to speak, on the point of whether according
to Ong, he would first kill the intended victim and demand money from his parents afterwards, or detain
him first and, after receiving a ransom money, kill the victim. Furthermore, from the first week of April, 1971,
when this intention was allegedly revealed by Ong to this witness, Ong could have changed his mind with
respect to the demand for money when the victim was actually taken and killed in the early morning of
April, 1971.

There was no Kidnapping to Make the Crime a Complex one of Kidnapping with Murder

The extrajudicial confession (Exhibit N) of accused Benjamin Ong was affirmed and confirmed by him in
open court, thus:

I show you this document marked as Exhibit "N", statement of Benjamin Ong, dated September
"Q
1,1971, do you admit that this is your statement given to the NBI?
A Yes, sir.

In this statement, Exhibit "N", you admitted that Henry Chua was taken from the Mustang car
and transferred to the Biscayne car and then brought to the uninhabited place in Barrio
Q
Makatipo, what was your purpose in having the late Henry Chua taken from his car and
brought to Makatipo?
A My purpose was just to kill him, and there is not going to be any delay.

Q Was there any purpose of detaining him for sometime?


A No, there was no purpose to detain him any further."
The evidence on record shows clearly that the deceased Henry Chua and Benjamin Ong left the Wigwam
Nightclub at Paraaque, at about 1:30 a.m. on April 24, 1971, in the car of Chua. Chua went voluntarily
with Ong, so much so that Chua himself drove his car. They were already in Del Monte Avenue, near the
place in Caloocan where Chua was killed and buried when they tied the hands of the deceased; that
there were still disagreement among the four accused oh who would kill the deceased, until finally it was
the co-accused Fernando Tan who stabbed him with an icepick; and that the four accused including two
others, parted from each other at 7:00 o'clock in the early morning of April 24, 1971 after they brought the
car of Chua and left it in Bo. Tibag, Balivag, Bulacan.

35
In view of the foreging facts and circumstances, We hold that there was no kidnapping, but only murder,
because the detention of Chua was only incidental to the main objective of murdering him and was not
a necessary means for the commission of the murder. From the Commentaries on the Revised Penal Code
of Justice Aquino, an acknowledged authority in criminal law, We find the following:

"If the detention of the victim is only incidental to the main objective of murdering him, and is not a
necessary means for the commission of the murder, the crime is only murder and not the complex one of
murder through kidnapping. In the Guerrero case, the accused Huks brought to the mountain two persons,
father and son. The father was killed. The son, a 14-year old minor, was able to escape on the second
night following his detention. HELD: The accused were guilty of murder as to the father and kidnapping
as to the son.

"In a 1902 case, the victim was taken from his house and then brought to an uninhabited place, where he
was murdered. HELD: The crime was murder only. There was no illegal detention 'since it does not appear
that it was the purpose of the accused to commit this offense.' The primary objective was to kill the victim:

"Where after the robbery committed in a house, three of its inmates were taken to a place near the river
one kilometer from the house, where they were killed, the kidnapping was deemed absorbed in the crime
Of robbery with homicide.

"Where the appellants kidnapped the victim at his house at Aviles Street, Manila and forced him to ride in
a car, but while the car was at the intersection of Libertad Street, Pasay City, the victim jumped from the
car and was shot to death, the crime was held to be murder only." (I Revised Penal Code by Justice
Aquino).

And We quote from the brief of appellant Ong:

"The crime committed was only murder.

"As early as the case of US vs. Nicolas Ancheta,.et al. (No. 422, March 14, 1902; 1 Phil. 165), it was held that
where the accused kidnapped the victim, Ventura Quinto, took him to a place called Radap, and there
by order of Nicolas Ancheta and Sebastian Dayag, the victim was killed, the crime committed by them
was murder. The acts committed by the accused do not constitute the crime of illegal detention since
the deceased was captured in his house and taken by the accused to an uninhabited place selected by
them for the purpose of killing them there. (At p. 169). In the case of US vs. Teodoro de Leon (No. 522),
March 10, 1902; 1 Phil. 163), there was a demand for the payment ransom. Nevertheless, the accused was
found guilty not of kidnapping with murder but of murder only. In this case, the deceased, Don Julio
Banson, was forcibly removed from his house by Fabian Tolome, by order of Teodoro de Leon. He was
tortured and maltreated by the defendant until they arrived at a place called Bulutong. 'Not satisfied with
torturing the deceased by himself he (Teodoro de Leon) ordered Tolome to give him a blow upon the
chest with a bolo. Don Julio begging for mercy, the defendant sent one of his servants to the wife of the
deceased to ask for $1,000.00 for his ransom. After the servant had been sent all were led to a place
called Cosme and upon arriving there the defendant ordered Fabian and Tomome to conduct Don Julio
to a ditch. At the same time the witness and his three companions were given their liberty by the
defendant, who remained with his two companions and with Don Julio. Don Julio was never afterwards
seen alive and his headless body was found two or three days later in this same place.' The accused was
found guilty of the crime of murder. Similarly, in the case of US vs. Emiliano Cajayon, et al. (No. 981, Oct. 8,
1903; 2 Phil. 570) twelve armed men kidnapped Tranquilino Torres and took him with them to the barrio
Maliig, in the town of Lubang, Cavite province, where they killed him and buried him in a hole dug for that
purpose. It was held that the crime committed was murder. The pertinent facts of the case are stated

36
briefly as follows: About 20 armed men forced their way into the house of Felix Marin, made him and his
son prisoners, and carried them off with their arms tied behind their backs. From there they proceeded to
the house of the head man of the barrio which they set on fire, and after capturing all the inmates, brought
them to an estero called the "Pasig" where they set all prisoners free, except Felix Marin and Isabel
Beltran. These two they took away in a boat and carried to a clump of manglares, at the edge of
the estero, where Maris, still bound, was decapitated by one of the band with a single stroke of a
bolo. Isabel Beltran was set free. It will be noted that as to Isabel Beltran, the son of Felix Maris and the
others, who were made prisoners, there was deprivation of liberty. Nevertheless, the accused was found
guilty of murder, and not of kidnapping with murder. In the case of People vs. Magno Quinto, et al. (L-
1963, Dec. 22, 1948; 82 Phil. 467), it was established that Gregorio Caling was picked up at his home in
Floridablanca, Pampanga by a band of Hukbalahap on the night of December 9, 1945 and taken to the
bank of the Gumain River, Gregorio Caling was investigated in connection with his arms, maltreated, and
subsequently killed. The judgment finding him guilty of murder was affirmed. In the case of People vs. Juan
Bulatao (L-2186, Jan. 29, 1949; 82 Phil. 743), one Jose Tan was forcibly taken by four armed men, among
them the accused. The following morning, the victim was found dead. It was also held that the accused
was guilty of murder. In the case of People vs. Eufracio Lansang (L-1187, Jan. 25, 1949; 82 Phil. 662) the
accused who participated in the kidnapping of the victim who was thereafter killed was found guilty as
an accomplice in the crime of murder. The case of People vs. Alejandro Mendiola, et al. (L-1642, 44, Jan.
29, 1959; 82 Phil. 740) is more significant. In this case the Supreme Court said:

'The circumstances of the case, as proved by the evidence, lead us to the conclusion that each and
everyone of appellant took part with Taciano V. Rizal in a conspiracy to kidnap as they did Teofilo Ampil
and they are all equally responsible for his killing, which was perpetrated in accordance with the plan of
the kidnappers. Once the kidnapping has been decided, the authors necessarily had to entertain the
killing as one of the means of accomplishing the purposes of kidnapping.

'The three appellants were correctly found by the trial court guilty as authors of the crime of murder. . .'
In the case of People vs. Francisco Moreno (L-2335, March 7, 1950; 85 Phil. 731), several armed men went
to the house of Manuel Artates in barrio Pagoncile, Aguilar, Pangasinan, and took him to the Marapudo
Mountains in Mangatarem where, he together with one Jose Jasmin, was beheaded. Thereafter, 'the
defendant Francisco cautioned all the men who took part in or witnessed the execution as well as the
kidnapping of the two men not to reveal to anyone what they had seen that night under penalty of
punishment.' The decision of the trial court finding the appellant guilty of murder was affirmed. In the case
of People vs. Alfredo Riparip, et al. (L-2408, May 31, 1950; 85 Phil. 526), one Enrique Roldan was on
December 27, 1944 kidnapped and on the following day killed by certain guerilla units. The accused were
found guilty of the crime of murder. In People vs. Gaudencio Villapa, et al. (L-4259, April 30, 1952; 91 Phil.
189, the deceased Federico Agonias, was taken by the accused from the house of Guillermo Calixto in
barrio San Marcelino, Balugao, Pangasinan, and he was killed about 50 meters from the house. They were
found guilty of murder. In People vs. Emeterio Sarata, et al. (L-3544, April 18, 1952; 91 Phil. 111), it appeared
that the four accused took the victim Sabiano Bucad from his house, placed him in a banca and sailed
towards the opposite shore of the Bato lake where the victim was maltreated and killed by the accused. It
was held that the crime committed was murder. In the case of People vs. Eligio Camo and Buenaventura
Manzanido (L-4741, May 7, 1952; 91 Phil. 240), the accused took the deceased Patricio Matundan from his
house in the barrio of Conda to the barrio of Talaan, both of the Municipality of Sariaya, Quezon. Upon
reaching a place near the mangroves, the group stopped, and accused Came shot and killed the
victim. The accused were charged with the crime of murder with kidnapping. The Supreme Court held:

'The Solicitor-General next contends that the offense committed was the complex crime of kidnapping
with murder. Again, we are inclined to agree with the trial court that the crime committed was simple
murder. It is true that Patricio was taken from his home but it was not for detaining him illegally for any
length of time or for the purpose of obtaining ransom for his release. In quite a number of cases decided

37
by this court where the victim was taken directly from his house to the place where he was killed,
kidnapping was not considered to raise the offense to the category of a complex.' (At p. 246)
In People vs. Nestorio Remalante (L-3512, Sept. 26, 1952; 92 Phil. 48), the accused with about 10 armed
men met Mercedes Tobias, accompanied by Eusebio Gerilla and Lucia Pilo, on the way to her home in
the barrio of Guiarona, Municipality of Dagami, Province of Leyte. The accused took hold of Mercedes
Tobias and dragged her, while at the same time striking her with the butt of his rifle at different parts of her
body. Eusebio Gerilla and Lucia Pilo saw Mercedes being dragged towards the sitio of Sawahan. Hardly
had they walked one kilometer when they heard gun reports. The following day, Mercedes was found
dead in Sawahan with two gunshot wounds. Nestorio Remalante was charged and found guilty by the
trial court of the crime of kidnapping with murder. As to the charge of kidnapping, the Supreme Court
held:

'There is no sufficient evidence of intention to kidnap because from the moment Mercedes Tobias was
held and dragged to the time when the gun reports were heard nothing was done or said by the appellant
or his confederates to show or indicate that the captors intended to deprive her of her liberty for sometime
and for some purposes and thereafter set her free or kill her. The interval was so short as to negative the
idea implied in kidnapping. Her short detention and ill-treatment are included or form part of the
perpetration of the crime of murder.' (at p. 51)
In the case of People vs. Silvino Guerrero, et al., (L-9559, May 14, 1958; 103 Phil. 1136, Unrep.), the appellants
were found guilty for the murder of Candido Disengano and the kidnapping of Paulo Disengano. As to
the killing of Candido Disengano, it was held:

As the court a quo has correctly held, appellants cannot be convicted of the complex crime of
kidnapping with murder under Article 48 of the Revised Penal Code, for the reason that kidnapping was
not a necessary means to commit the murder. Candido was detained and brought to the mountains to
be killed - this we have held may not be considered kidnapping with murder but mere murder. (People
vs. Camo, G.R. No. L-4741, May 7, 1952; People vs. Remalante, G.R. No. L-3512, 48 O.G. 3881-3883; People
vs. Villapa, et al.,G.R. No. L-4259, April 30, 1952) [13 Velayo's Digest (new Series) 337; please see also 103
Phil. 1136]'
In People vs. Santos Umali, et al. (L-8860-70, January 23, 1957; 100 Phil. 1095 Unrep.), the accused were
charged and convicted by the trial court of kidnapping with murder. The evidence shows that the
deceased was killed in front of his house. The crime committed is only murder. (13 Velayo's Digest [New
Series], p. 340).

In People vs. Cenon Serrano alias Peping, et al. (L-7973, April 27, 1959; 105 Phil. 531), the accused were
charged with illegal detention with murder. After a drinking spree, the accused, Cenon Serrano,
suggested to the deceased Pablo Navarro to leave Bacolor, Pampanga for San Fernando for a good
time, to which suggestion the latter agreed. While the victim together with the accused Cenon Serrano
and others were on the way to San Fernando, Cenon Serrano suggested that they proceed to Angeles for
a good time to which Pablo Navarro agreed. Upon reaching barrio San Isidro, Cenon Serrano ordered
the driver to proceed to barrio Dolores, Bacolor, Pampanga where the deceased was detained and
questioned at the stockade of the civilian guards. That same afternoon, Pablo Navarro was taken out of
the stockade and was brought to sitio Castilang Malati where the deceased was shot and killed. The trial
court found the defendants guilty of the crime of murder. The decision was affirmed by the Supreme
Court. In People vs. Rosario Lao, et al. (L-10473, January 28, 1961; 1 SCRA 42), one Rosa Baltazar was taken
by two of the accused and killed beside a creek about 6 to 10 meters away from the hatchery of the Lao
poultry farm where she was staying. The trial court found them guilty of the crime of kidnapping with
murder. The Supreme Court held that 'the crime committed is not kidnapping with murder as stated in the
title of the information but murder.'

38
In People vs. Felipe Sacayanan (L-15024-25, Dec. 31, 1960; 110 Phil. 588), a group of five armed men forcibly
took from their houses the victims Juan Galaraga and Victor Alamar to a place about 40 meters away
from the house where they were shot. Juan Galaraga died. Victor Alamar was seriously wounded. The
trial court convicted the accused of the complex crime of kidnapping with murder. The Supreme Court
held that this was error. 'Nothing was said or done by the accused on his confederates to show that they
intended to deprive their victims of their liberty for some time and for some purpose. There was no
appreciable interval between their being taken and their being shot from which kidnapping may be
inferred.' (See People vs. Remalante, 92 Phil. 48; O.G. [9] 38881).

From the foregoing discussion, it seems clear that the weight of authority is in favor of the proposition that
where the victim was taken from one place to another, solely for the purpose of killing him and not for
detaining him for any length of time or for the purpose of obtaining ransom for his release, the crime
committed is murder, and not the complex crime of kidnapping with murder. This ruling is entirely consistent
with the law. Art. 267 of the Revised Penal Code penalizes a person 'who shall kidnap or detain another,'
and the penalty becomes capital 'where the kidnapping or detention was committed for the purpose of
extorting ransom from the victim or any other person.'

xxx xxx xxx

In the case at bar, the only evidence appreciable against the appellant Benjamin Ong regarding the
surrounding circumstances of Henry Chua's death are (1) the extrajudicial statement of Benjamin Ong, (2)
the testimony of Benjamin Ong during the trial, (3) the testimony of agent Enrique Lacanilao about the
reenactment of the crime.

In the extrajudicial statement (Exhibit N) Benjamin Ong said that from the Wigwam nightclub, Henry Chua
and he rode on Henry's Mustang car with the latter driving it. Fernando Tan and his friend were in the
Biscayne car of Benjamin Ong following the Mustang. (Answer to Question No. 40, p. 3, Exh. N). At Araneta
Avenue in Quezon City, Benjamin Ong requested Henry Chua to stop the car to enable him to
urinate. When Henry Chua complied, Fernando Tan and his friend stopped in front of the Mustang car,
pretending to be policemen, and ordered Henry Chua to go with them to the police precinct. (Id., p. 5)
Fernando Tan drove the Biscayne car, while Benjamin Ong in Henry Chua's car followed. From Araneta
Avenue, Fernando Tan drove to Novaliches where Henry Chua was killed. (Id.) It will be noted that no
appreciable time elapsed from arrival at Novaliches up to the time Henry Chua was killed, to indicate a
separate intention to deprive the latter of his liberty. When Benjamin Ong testified on September 22, 1971,
he affirmed his admission of responsibility for the death of Henry Chua (t.s.n., Sept. 22, 1971, p. 26). He
further testified as follows:

ATTY. QUISUMBING:

In this statement Exhibit "N", you admitted that Henry Chua was taken from the mustang car
Q and transferred to the biscayne car and then brought to that uninhabited place in having the
late Henry Chua taken from his car and brought to Makatipo?

A My purpose was just to kill him, and there is not going to be any delay.

Q Was there any purpose of detaining him for sometime?

xxx xxx xxxx

39
A No, there was no purpose to detain him any further. (Id., pp. 27-28)
The narration of agent Enrique Lacanilao about the re-enactment of the crime showed that there was no
detention of the deceased Henry Chua for any length of time. He was killed and promptly buried. (Please
see pp. 43-47, t.s.n., Sept. 18, 1971). On the basis of the foregoing evidence, the accused can hardly be
held liable for kidnapping as well. It may not be amiss to state that an accused is entitled to acquittal
unless his guilt is shown by proof beyond reasonable doubt. (Rule 133, Section 1, Revised Rules of
Court). The evidence at hand hardly satisfied the requirement of proof beyond reasonable doubt as to
the charge of kidnapping. The necessary result is that the accused can be held liable only for the killing
of Henry Chua." [Brief for the Appellant Benjamin Ong y Kho, pp. 43 to 56]
And the evidence on record clearly show that Henry Chua voluntarily went with Benjamin Ong when they
left the Wigwam Nightclub at Paraaque at about 1:30 a.m. on April 24, 1971, so much so that they rode
in the car of Chua and it was driven by Chua himself. The two drove straight down Roxas Boulevard, then
to Quiapo, and Quezon Boulevard Extension in Quezon City; and after passing Sto. Domingo Church, they
made a turn towards a dirt road leading to Del Monte Avenue. When they reached a dark and secluded
place, Benjamin Ong urged Chua to stop the car for the former to urinate to which the latter obliged. The
Biscayne car where Fernando Tan, Bienvenido Quintos and Baldomero Ambrosio were riding,
stopped. Fernando Tan poked his gun at Chua and pulled him down from his Mustang car with Ambrosio
giving help. His hands were tied, his mouth gagged with a flannel cloth, and he was placed in the Biscayne
car. Tan and Bienvenido Quintos then rested their feet on him. Then Ambrosio drove the Biscayne while
Ong drove the Mustang. They proceeded towards Barrio Makatipo, Novaliches, Caloocan City, where
Henry Chua was stabbed to death and buried.

In other words, the time interval when the deceased Henry Chua was actually deprived of his liberty was
short (from Del Monte Avenue to Barrio Makatipo, Novaliches, Caloocan); and the same was
only incidental to the main objective of murdering him.

The only authority cited by the prosecution on this point is that of the case of Parulan vs. Rodas (88 Phil.
615). But the ruling in the Parulan case cannot be applied to the case at bar, because in the Parulan
case, the Court found that the kidnapping was a necessary means for the purpose of extorting ransom
from the victim and killing him if the desired amount could not be given; and that the defendants had to
kidnap or carry the victim from Manila (where he was already deprived of his liberty, with Parulan poking
his gun on the victim), to a faraway and secluded place (a river in Bambang, Bulacan) in order to better
secure the consent of the victim through fear to pay the ransom, and kill him if he refuses to accede to
their demands, as in fact he was killed by Parulan because of his (victim's) refusal to pay the ransom.

We Hold that Both Appellants are Guilty of Murder

The killing of the victim in this case was attended by several qualifying and aggravating
circumstances. The facts on record prove this, beyond reasonable doubt, even if we were to disregard
the extrajudicial confession of Benjamin Quintos which he denied and was allegedly extracted from him
through force and intimidation.

Treachery (alevosia) qualified the killing to murder. Undisputed facts show that Henry Chua's hands were
tied and his mouth was gagged with a flannel cloth before he was stabbed twice with an icepick and
buried in a shallow grave near a creek. These facts portray well that the tied hands of the victim rendered
him defenseless and helpless thereby allowing the accused to commit the crime without risk at all to their
person.[50]

40
The accused Benjamin Ong and Bienvenido Quintos, however, were quick to insist that this circumstance
should not be taken against them because they did not do the actual stabbing (which was done by
Fernando Tan). Easily, the weakness of this claim can be discerned. Conspiracy, connivance and unity of
purpose and intention among the accused were present throughout in the execution of this crime. The
four participated in the planning and execution of the crime and were at the scene in all its stages. They
cannot escape the consequence of any of their acts even if they deviated in some detail from what they
originally thought of. Conspiracy implies concert of design and not participation in every detail of
execution.[51] Thus, treachery should be considered against all persons participating or cooperating in the
perpetration of the crime.[52]

With regards to the aggravating circumstance of abuse of superior strength, the same should be deemed
absorbed in treachery. This position is itself supported by the Acting Solicitor General in his brief and is
sustained in a long line of decisions.[53]

In the same vein, the accused would like the aggravating circumstance of nighttime (nocturnidad) to be
absorbed in treachery in that it forms part of the peculiar treacherous means and manner adopted to
insure the execution of the crime. The case of People vs. Berdida[54] provides the exception to this rule and
is applicable to the case at bar. It was there held that:

"From the facts and evidence of record in this case, it is clear that appellants took advantage of nighttime
in committing the felonies charged. For it appears that to carry out a sentence they had pronounced
upon Antonio Maravilla and Federico Canalete for the death of one Pabling, they had evidently chosen
to execute their victims under the cover of darkness, at the dead of night, when the neighborhood was
asleep. Inasmuch as the treachery consisted in the fact that the victim's hands were tied at the time they
were beaten, the circumstance of nighttime is not absorbed in treachery, but can be perceived distinctly
therefrom, since the treachery rests upon an independent factual basis. A special case therefore is present
to which the rule that nighttime is absorbed in treachery does not apply."[55]
This aggravating circumstance was correctly appreciated by the lower court regardless of whether or not
the same was purposely and deliberately sought by the accused for it is clear that the darkness of the
night facilitated the commission of the crime and was taken advantage of by them.[56]

The purposive selection of an uninhabited place (despoblado) is likewise clear from the evidence. The
killing was done in Barrio Makatipo, Novaliches, Caloocan City, an isolated place that resembled that of
an abandoned subdivision. The place was ideal not merely for burying the victim but also for killing him
for it was a place where the possibility of the victim receiving some help from third persons was completely
absent. The accused sought the solitude of the place in order to better attain their purpose without
interference, and to secure themselves against detection and punishment.[57] As aptly stated in the
"Sentence" of the lower court:

". . . The possibility of the victim calling for succor or assistance from any third person was ruled out by the
chosen site. Trees, lush vegetation and thick cogon grasses hide the place where the crime was
committed from the view of even a chance passerby. The choice of an uninhabited place for the killing
of Henry Chua, therefore, further aggravated the offense committed by the accused. People vs. Curiano,
L-15256-57, October 31,1962; U.S. vs. Vitug, 17 Phil. 1)."[58]
In the case of the aggravating circumstance of abuse of confidence (abuso de confianza), it appears
that the lower court wrongly appreciated this circumstance. In order for this circumstance to obtain, it is
necessary that there be a relation of trust and confidence between the accused and the one against
whom the crime was committed, and that the accused made use of such relation to commit the
crime,[59] It is essential too that the confidence be a means of facilitating the commission of the crime, the
culprit taking advantage of the offended party's belief that the former would not abuse said
confidence.[60]

41
Nowhere in the records does it appear that Henry Chua reposed confidence upon the person of Benjamin
Ong. If any, Henry Chua was simply not afraid of Benjamin Ong, having told and bragged to the latter
about his violent exploits in the past and threatened him with bodily harm in case of failure to pay.[61] He
knew that he was far stronger than Benjamin Ong in terms of influence and money. He thought that
Benjamin Ong would fear him. The fact that Henry Chua invited Ong for nightclubbing that fatal evening
and accommodated him in his car on their way home from the nightclub does not mean that Henry Chua
had confidence in him. There was no special relation of confidence between them. He knew that
Benjamin owed him a substantial amount and that its settlement had long been overdue which fact
irritated him very much. Benjamin Ong and Henry Chua were together that night in the nightclub as well
as in the car not because of said confidence. It was simply because Benjamin Ong had some accounts
to settle with him. Thus, in the case of U.S. vs. Cruz, et al.,[62] it was held that:

"...The fact of Cabaya having simulated friendship and desire for work, together with the companions who
went with him, and the fact that he received food and work immediately upon being accepted by the
Americans to work in the mines, is not, as stated in the judgment, a degree of treachery, according to law,
sufficient to constitute the aggravating circumstance of abuse of confidence. It may however, be argued
as unworthy conduct and ingratitude, but not as abuse of confidence. It is necessary first to show what
has been the confidence granted or given in order to determine whether there was or was not an abuse
of it, and in the present case there is nothing to show what the confidence given or conceded to Cabaya
was, that could facilitate the commission of the crime."
Likewise, in the case of People vs. Brocal,[63] it was held that:

"There is no abuse of confidence in attempted rape where on the day of the crime the accused was in
the company of the offended girl, not because of her confidence in him, but because they were partners
in a certain business."
More convincing this time is the aggravating circumstance of use of motor vehicle in the commission of
the crime. The Biscayne car of Benjamin Ong was used in trailing the victim's Mustang car from Wigwam
Nightclub up to the time that it was overtaken and blocked. It carried the victim on the way to the scene
of the killing; it contained at its baggage compartment the pick and shovel used in digging the grave; it
was the fast means of fleeing and absconding from the scene. Again, the motor vehicle facilitated the
stark happening. It has been held that the use of motor vehicle is aggravating in murder where the said
vehicle was used in transporting the victim and the accused.[64]

Cruelty (ensaamiento), as an aggravating circumstance, cannot be considered here. The brief of the
Acting Solicitor General agrees with that of the accused in denying the attendance of cruelty as an
aggravating circumstance. Indeed, as it appears from the record, the group intended merely to kill the
victim, bury him, and flee from the locale of the fearful crime. For cruelty to exist, it must be shown that the
accused enjoyed and delighted in making their victim suffer slowly and gradually, causing him
unnecessary physical or moral pain in the consummation of the criminal act.[65] Even granting that the
victim died because of asphyxiation when he was buried and not hemorrhage from stab wounds, as
testified to by Dr. Ibarrola,[66] which, however, has been contradicted by his own necropsy report which
shows that the cause of death was the "punctured wounds in the abdomen," and by Dr. Lara who testified
that the two wounds could have produced death due to shock, it appears that the victim's burial was not
meant to make him suffer any longer but simply to conceal his body and the crime itself.

Concededly, the qualifying circumstance of evident premeditation (premeditacion conocida) attended


the commission of the crime. What else can better portray this circumstance than the frequent
meetings[67] of the four accused at the Barrio Fiesta Restaurant in order to discuss, lay out the plan, and
secure the different paraphernalia consisting of the rope, icepick, flannel cloth, flashlight and
shovel.[68] Added to this is the careful selection of an "ideal" site for the grissly happening. [69] Similarly, the

42
plan to go to Taipeh and Hongkong immediately after the incident pictures the presence of evident
premeditation.[70] The accused meditated and tenaciously persisted in the accomplishment of the crime
and were not prompted merely by the impulse of the moment.[71]

Although the claim of the accused Benjamin Ong that the mitigating circumstance of plea of guilty should
be appraised in his favor appears academic herein, as will be shown in the dispositive portion, for the sake
of argument, however, the same is given weight here. Indeed, the kidnapping portion of the crime cannot
be appreciated here beyond reasonable doubt as stated at the outset. Furthermore, it can be seen that
the prosecution alleged so many aggravating circumstances which should be absorbed in one or the
other. To plead guilty to this information naturally would be most unfair for the accused especially where
the penalty would be the capital punishment of death. The accused showed signs of remorsefulness upon
his arrest when he cooperated with the police authorities in the solution of the crime. As held in the case
of People vs. Yturriaga,[72]

"x x x It only remains to consider briefly whether the defendant's plea of guilty in the form it was entered
constitutes a voluntary confession of guilt before the court as defined in the same subsection of Article
13. We think it does.

"Although the confession was qualified and introduction of evidence became necessary, the qualification
did not deny the defendant's guilt and, what is more, was subsequently fully justified. It was not the
defendant's fault that aggravating circumstances were erroneously alleged in the information and
mitigating circumstances omitted therefrom. If such qualification could deprive the accused of the
benefit of plea of guilty, then the prosecution could nullify this mitigating circumstance by counteracting
it with unfounded allegations of aggravating circumstances."
We hold that the accused Benjamin Ong is likewise entitled to the mitigating circumstance that is
analogous to passion and obfuscation (Art. 13, par. 10, Revised Penal Code), based on the following facts
stated in his brief:

a) Henry Chua and his companions went to the office of Benjamin Ong. In a loud voice, with angry
gestures, and in the presence of his subordinates and fellow employees, Henry Chua demanded payment,
and threatened bodily harm to him and his family.

b) Henry Chua went as far as to threaten the life of Benjamin Ong unless his obligation to Chua was
paid. "If you treasure your life, you better pay first."

c) Because of this incident, he, Benjamin Ong, "was humiliated."

d) His brother-in-law, Chua Pak, told him that he was holding a very responsible position in the company
and so he should not be involved in any scandal.

e) He was "discredited and degraded in front of my brother-in-law. He was so embarrassed, he finally


tendered his resignation from the company.

f) Because of the threat of Henry Chua, the accused tried to get money from all sources but he was not
successful. The allotted time was so short. To relieve him of the pressure brought to bear upon him to pay
his gambling debt, he even thought of embezzling money belonging to the company in which he worked.

g) Because of his inability to raise money to be paid to Henry Chua, he became "deeply depressed." He
felt: "I was being turned into a criminal.

h) He begged Henry Chua to give him more time to raise the money. "Nagmamakaawa na ako sa

43
kanya." This was the night before Henry Chua was killed. If Henry Chua had granted him time "the whole
plan to kill Henry Chua might not materialize." But Henry Chua, while not relenting, but perhaps in utter
contempt and disdain of Benjamin Ong instead decided to transfer from Amihan to Wigwam because he
wanted to be entertained by a hostess. Henry Chua, it will be noted, was well known to Wigwam hostess,
Ligaya Tamayo. Benjamin Ong was seen by her for the first time that evening.

i) So while Chua enjoyed himself, Benjamin Ong was worried, as he pleaded with Henry Chua in vain for
more time to pay the obligation.

xxx xxx xxx


In People vs. Timoteo Olgado, et al. (L-4406, March 31, 1952; 91 Phil. 908, Unrep.), the two accused were
provoked to commit two murders because of the indecent propositions made to the women by Jalumio
and his companions. For Mario Aninias, this is the mitigating circumstance of passion and obfuscation or
vindication of a grave offense to his wife.[73

In this regard, accused Benjamin Ong filed on October 10, 1973 before this Court a Petition for New Trial
and/or to Consider Case as Simple Murder.[74]In this petition, Benjamin Ong's wife, Athena Caw Siu Tee
Ong, alleged in an affidavit an incident when her husband refused to allow her to testify on during the
regular trial in the lower court. She said that Benjamin Ong suppressed it became it would be a source of
"great shame" to their family. Indeed, the records show how Benjamin Ong's counsel vainly convinced him
to tell it but he refused to do so.[75] Lately, Benjamin Ong has changed his mind and has consented to his
wife's divulging the story. Said story simply consists of Henry Chua's proposal of love and attempted rape
allegedly committed on the person of Athena on April 15, 1971 which Henry Chua asked in lieu of the
payment of the gambling debt. However, this matter is now academic because it would only tend to
bolster the mitigating circumstance that is analogous to passion and obfuscation, which we have just
considered in favor of the accused Benjamin Ong.

IN VIEW OF ALL THE FOREGOING, the two accused-appellants Benjamin Ong y Kho and Bienvenido Quintos
y Sumaljag, are hereby found guilty beyond reasonable doubt of the crime of murder with the attendant
qualifying circumstance of treachery, and the aggravating circumstances of evident premeditation and
use of motor vehicle. These two circumstances are offset by the mitigating circumstances of plea of guilty
and one similar or analogous to passion or obfuscation which are appreciated in favor of accused-
appellant Benjamin Ong who is hereby sentenced to reclusion perpetua. Justices Teehankee and
Makasiar, however, are of the opinion that the crime committed by the two accused-appellants Benjamin
Ong and Bienvenido Quintos is kidnapping with murder and that the kidnapping was conceived for the
purpose of extorting ransom, among other motives. The members of the Court failed to arrive at a clear
consensus on the existence of the aggravating circumstances of "nighttime" and "uninhabited place"
(which Justice Barredo, in his concurring and dissenting opinion, concluded do not obtain in this case).

With respect to the accused-appellant Bienvenido Quintos, although no mitigating circumstance can be
appreciated in his favor, and he should therefore be sentenced to death, the Court hereby imposes upon
him the penalty of reclusion perpetua and not death, because of Our conclusion that his co-accused-
appellant Benjamin Ong should be sentenced only to reclusion perpetua, and because Justice Barredo,
in his concurring and dissenting opinion, even concluded that Bienvenido Quintos is guilty only as an
accomplice; and hence, in any event, We would not have the necesssary ten votes for the imposition of
the death penalty upon said accused-appellant.

As We hereby sentence the two accused-appellants Benjamin Ong and Bienvenido Quintos to suffer the
penalty of reclusion perpetua, We affirm that part of the decision under review, which sentenced them
jointly and severally to indemnify the heirs of the deceased Henry Chua in the amount of P12,000.00; to

44
pay moral damages in the amount of P50,000.00, and another P50,000.00 as exemplary damages; and to
pay their proportionate share of the costs, as We find no reason to disturb the same.

PEOPLE OF THE PHILIPPINES, PLAINTIFF-APPELLEE,


~vs~
RICKY ARGUTA ALIAS JOEL & WILSON CAHIPE ALIAS SIWIT, ACCUSED-APPELLANTS.

DECISION
PERLAS-BERNABE, J.:
Before the Court is an ordinary appeal[1] filed by accused-appellants Ricky Arguta alias Joel (Arguta)
and Wilson Cahipe alias Siwit (Cahipe; collectively, accused-appellants) assailing the Decision[2] dated
April 24, 2014 of the Court of Appeals (CA) in CA-G.R. CEB-CR HC No. 01462, which affirmed with
modification the Decision[3] dated July 25, 2008 of the Regional Trial Court of Tacloban City, Branch 6 (RTC)
in Crim. Case Nos. 97-02-76 and 97-02-77 finding accused-appellants guilty beyond reasonable doubt of
one (1) count of Rape, defined and penalized under the Revised Penal Code (RPC), as amended.
The Facts
On January 30, 1997 two (2) criminal informations were filed before the RTC charging Cahipe with two (2)
counts of Rape, and Arguta of one (1) count of the same crime, viz.:
Crim. Case No. 97-02-76
That on or about the 5th day of December 1996 in the Municipality of Tanauan, Province of Leyte,
Philippines and within the Jurisdiction of this Honorable Court, the above-named [accused-
appellants], conspiring, confederating and mutually helping each other, motivated by lewd
design, with the use of a bladed weapon, by means of force and intimidation, did then and
there willfully, unlawfully and feloniously, have carnal knowledge of [AAA],[4] without her consent
and against her will.
Contrary to Law.
Tacloban City, January 30, 1997.
Crim. Case No. 97-02-77
That on or about the 5th day of December 1996, in the Municipality of Tanauan, Province of Leyte,
Philippines, and within the jurisdiction of this Honorable Court, the above-named accused
[Cahipe], motivated by lewd design, by means of force and intimidation, did then and there
willfully, unlawfully and feloniously, have carnal knowledge of [AAA], without her consent and
against her will.
Contrary to Law.
Tacloban City, January 30, 1997.[5]
According to the prosecution, at around 8 oclock[6] in the evening of December 5, 1996, AAA was
instructed by her father to fetch her sister in school. However, AAA failed to find her sister and decided to
go back home. On her way home, accused-appellants intercepted AAA, threatened her with a bladed
weapon, dragged her to a cottage at a nearby beach resort, and bound her hands and feet. Thereafter,
they removed her clothes and placed her on the floor. Arguta then mounted AAA and inserted his penis
into her vagina. After Arguta satisfied his lust, Cahipe took over and raped her. Thereafter, accused-
appellants left AAA at the cottage. An hour later, Cahipe returned and dragged AAA to a store owned
by a certain Lino Ostero[7] (Ostero). There Cahipe undressed her again, mounted her, and inserted his penis

45
into her vagina. Afterwards, AAA was returned to the cottage. The next day, AAAs father found her crying
at the cottage.[8]
Further, the prosecution offered the findings of the physical examination by a certain Dr. Eilleen Colaba on
AAA, stating, inter alia, that: (a) AAAs genitalia was grossly normal, which means no abnormality; (b) AAA
has complete healed hymenal lacerations at the 5 oclock and 7 oclock positions and a partially healed
hymenal laceration at the 12 oclock position; and (c) AAAs genitalia is negative for the presence of
spermatozoa.[9]
In their defense, accused-appellants both denied the accusations leveled against them, and offered their
respective alibis. Cahipe claimed that on the date and time of the alleged incident he was minding
Osteros store. On the other hand, Arguta averred that he was at Osteros house watching television during
the time that the incident supposedly occurred. They both asserted that they did not know why AAA would
accuse them of raping her.[10]
The RTC Ruling
In a Decision[11] dated July 25, 2008, the RTC found accused-appellants guilty beyond reasonable doubt
of the crime of Simple Rape in Crim. Case No. 97-02-76 and, accordingly, sentenced them to suffer the
penalty of reclusion perpetua and ordered them to pay AAA, jointly and severally, the amounts of
P50,000.00 as civil indemnity and P50,000.00 as moral damages. Further, the RTC found Cahipe not guilty
of the crime of Rape in Crim. Case No. 97-02-77 and, accordingly, acquitted him due to insufficiency of
evidence.[12]
In finding the guilt of accused-appellants, the RTC held that AAAs testimony, as well as the medico-legal
report, established that on December 5, 1996, accused-appellants intercepted AAA, threatened her with
a bladed weapon, dragged her to a nearby cottage, undressed her, bound her, and took turns raping
her. The RTC did not lend credence to accused-appellants defense of denial and alibi, in light of the
positive assertions made by AAA, and considering that it was not physically impossible for them to have
been at the place of the crime on the date of the incident.[13]
However, as regards the second count of Rape against Cahipe, the RTC opined that it would be unusual
for AAA, who had just been raped and left alone in the cottage, to not attempt to escape or shout for
help when she was being transported to Osteros store and back to the cottage, observing that AAA had
to pass Osteros house before reaching the latters store. According to the RTC, these pose serious doubts
as to the existence of the second rape charge, thus, necessitating its dismissal.[14]
Dissatisfied, accused-appellants appealed their conviction to the CA.
The CA Ruling
In a Decision[15] dated April 24, 2014, the CA affirmed accused-appellants conviction with modification
ordering the accused-appellants to jointly and severally pay AAA the amount of P30,000.00 as exemplary
damages, in addition to the other amounts already awarded, and imposed interest at the rate of six
percent (6%) per annum on all the monetary awards from the date of finality of its Decision until fully
paid.[16]
Agreeing with the RTCs findings, the CA ruled that AAAs categorical and straightforward testimony
prevailed over accused-appellants denial and alibi. It observed that accused-appellants were in the
vicinity of the locus criminis at the time of the incident, and that the two could easily reach the cottage
where the rape occurred.[17] Thus, it concluded that accused-appellants actions fell squarely within the
definition of Rape under Article 266-A of the RPC, noting that accused-appellants had carnal knowledge
of AAA, and such was attained through force, threat, or intimidation.[18]
Aggrieved, accused-appellants filed the instant appeal.
The Issue Before the Court
The issue for the Courts resolution is whether accused-appellants conviction for Rape should be upheld.

46
The Courts Ruling
The appeal is bereft of merit.
At the outset, it must be stressed that in criminal cases, an appeal throws the entire case wide open for
review and the reviewing tribunal can correct errors, though unassigned in the appealed judgment, or
even reverse the trial courts decision based on grounds other than those that the parties raised as
errors.[19] The appeal confers upon the appellate court full jurisdiction over the case and renders such court
competent to examine records, revise the judgment appealed from, increase the penalty, and cite the
proper provision of the penal law.[20] Proceeding from the foregoing, the Court deems it appropriate to
modify accused-appellants conviction from Simple Rape to Qualified Rape, as will be explained
hereunder.
In this case, the Court notes that the rape occurred during the effectivity of the old rape provision of the
RPC, i.e., Article 335,[21] and, thus, the latter provision is controlling in this case, to wit:
Art. 335. When and how rape is committed. Rape is committed by having carnal knowledge
of a woman under any of the following circumstances:
1. By using force or intimidation;
2. When the woman is deprived of reason or otherwise unconscious; and
3. When the woman is under twelve years of age or is demented.
The crime of rape shall be punished by reclusion perpetua.
Whenever the crime of rape is committed with the use of a deadly weapon or by two or more
persons, the penalty shall be reclusion perpetua to death.
x x x x (Emphasis and underscoring supplied)
Under this provision, the elements of Rape are: (a) the offender had carnal knowledge of the victim; and
(b) said carnal knowledge was accomplished through the use of force or intimidation; or the victim was
deprived of reason or otherwise unconscious; or when the victim was under twelve (12) years of age or
demented.[22] The provision also states that if the act is committed either with the use of a deadly weapon
or by two (2) or more persons, the crime will be Qualified Rape, necessitating the imposition of a higher
penalty.[23] In People v. Lamberte,[24] the Court clarified the legal effect of the presence of both
circumstances, as follows:
The presence of either circumstance use of a deadly weapon or by two or more persons
qualifies the crime. If one is present, the remaining circumstance, if also attendant, is not a
generic aggravating circumstance. That was our ruling in People vs. Garcia, [192 Phil. 311, 342]
(1981) reading:
In the prosecution of the cases at bar, two circumstances are present, namely. 1. use of
a deadly weapon and 2. that two persons committed the rapes. The first was alleged in
the information while the second was proved during trial. In both cases, the Court
appreciated the first as a qualifying circumstance and the second as a generic
aggravating circumstance, in accordance with settled jurisprudence according to the
trial court.
We do not agree. Under the law above quoted, either circumstance is qualifying. When
the two circumstances are present, there is no legal basis to consider the remaining
circumstance as a generic aggravating circumstance for either is not considered as such
under Article 14 of the Revised Penal Code enumerating what are aggravating
circumstances. Hence, the correct penalty is the lesser penalty, which is reclusion
perpetua, there being no aggravating or mitigating circumstance, pursuant to Article 63,
paragraph 2, No. 2, Revised Penal Code,[25] (Emphases and underscoring supplied)

47
In this case, records reveal that accused-appellants threatened AAA with a bladed instrument and tied
her up before having carnal knowledge of her without her consent. Jurisprudence holds that force or
intimidation, as an element of Rape, need not be irresistible; as long as the assailants objective is
accomplished, any question of whether the force employed was irresistible or not becomes irrelevant.
Intimidation must be viewed from the lens of the victims perception and judgment and it is enough that
the victim fears that something will happen to her should she resist her assailants advances. [26] In this
regard, case law provides that the act of holding a bladed instrument, by itself, is strongly suggestive of
force or, at least, intimidation, and threatening the victim with the same is sufficient to bring her into
submission.[27]
In view of the foregoing, the Court finds no reason to deviate from the findings of fact made by the courts a
quo that accused-appellants are guilty as charged, i.e., of raping AAA with the use of a deadly weapon,
as the same are supported by the records. It must be noted that the assessment and findings of the trial
court are generally accorded great weight, and are conclusive and binding to the Court if not tainted
with arbitrariness or oversight of some fact or circumstance of weight and influence, [28] as in this case.
Nevertheless, considering that the crime was committed by two (2) persons, the accused-appellants
herein, with the use of a bladed weapon, it is only appropriate to increase their conviction from Simple
Rape to Qualified Rape.
Anent the proper penalty to be imposed, Section 3 of Republic Act No. 9346[29] provides that [p]ersons
convicted of offenses punished with reclusion perpetua, or whose sentences will be reduced to reclusion
perpetua, by reason of this Act, shall not be eligible for parole under Act No. 4103, otherwise known as the
Indeterminate Sentence Law, as amended. Pursuant thereto, accused-appellants should be sentenced
with the penalty of reclusion perpetua, without eligibility for parole.[30]
Finally, to conform with prevailing jurisprudence, the Court increases the award of damages in favor of
AAA to the amounts of P75,000.00 as civil indemnity, P75,000.00 as moral damages, and P30,000.00 as
exemplary damages, with six percent (6%) legal interest per annum on all the monetary awards from the
date of finality of judgment until fully paid.[31]
WHEREFORE, the appeal is DENIED. The Decision dated April 24, 2014 of the Court of Appeals in CA-G.R.
CEB-CR HC No. 01462 is hereby AFFIRMED, finding accused-appellants Ricky Arguta alias Joel and Wilson
Cahipe alias Siwit (accused-appellants) GUILTY beyond reasonable doubt of the crime of Qualified
Rape as defined and penalized under Article 335 of the Revised Penal Code
with MODIFICATION sentencing accused-appellants to suffer the penalty of reclusion perpetua, without
eligibility for parole, and ordering them to jointly and severally pay AAA the amounts of P75,000.00 as civil
indemnity, P75,000.00 as moral damages, and P30,000.00 as exemplary damages, with legal interest at the
rate of six percent (6%) per annum on all the monetary awards from the date of finality of this Decision until
fully paid.
SO ORDERED.
Sereno, C. J., (Chairperson), Leonardo-De Castro, Bersamin, and Perez, JJ., concur.

[G.R. No. 137050. July 11, 2001]

PEOPLE OF THE PHILIPPINES, plaintiff-appellee, vs. GEORGE CORTES y ORTEGA, accused-appellant.

DECISION
PARDO, J.:

48
The case is before the Court on automatic review of the decision[1] of the Regional Trial Court, Surigao
del Sur, Branch 29, Bislig, finding accused George Cortes y Ortega guilty beyond reasonable doubt of
murder and sentencing him to the supreme penalty of death.
On August 12, 1998, provincial prosecutor Alfredo J. Pondoc of Surigao del Sur filed with the Regional
Trial Court, Surigao del Sur, Branch 29, Bislig, an Information for murder against accused George Cortes y
Ortega, which reads as follows:

That on or about 11:00 oclock in the evening, more or less, of June 24, 1998, at P. Lindo Street, Saint Paul
District, Nangagoy, Bislig, Surigao del Sur, Philippines and within the jurisdiction of this Honorable Court, the
above-named accused with treachery and evident premeditation, armed with a knife and with intent to
kill did then and there willfully, unlawfully and feloniously attacked, assault and stabbed one Edlyn S.
Gamboa, a 16 year old girl, thereby inflicting the latter multiple stab wounds on her body which caused
her instantaneous death as certified by the doctor, to the damage and prejudice of the victims heirs.

Contrary to law: In violation of Article 248 of the Revised Penal Code.[2]

On June 24, 1998, at about eleven oclock in the evening, Junilla Macaldo was sitting on a bench
outside her house located at P. Lindo St., Saint Paul District, Mangagoy, Bislig, Surigao del Sur. While thus
seated, Edlyn Gamboa came to her asking for the whereabouts of Yen-yen Ibua. Junilla noticed that Edlyn
was followed by accused George Cortes. Junilla then instructed Edlyn to go upstairs of the house. When
Edlyn complied, accused followed her and successively stabbed her several times. Junilla tried to help
Edlyn, but accused overpowered her. In a moment, Edlyn was able to run away despite being wounded;
however, she collapsed five (5) meters away from where she was stabbed. Junilla shouted for help. At this
juncture, accused scampered away. Edlyn was able to stand up but again collapsed after walking about
five (5) steps. She was brought to the Babano Medical Clinic, where she expired.
Accused admitted that he stabbed Edlyn. He mistook Edlyn for her male companion against whom
he had an altercation earlier. He committed the mistake because at the time of the incident, accused
was very drunk and the place was very dark. He only learned that he had stabbed the wrong person the
following morning through the radio vigilantes program.
On August 28, 1998 the trial court arraigned the accused.[3] He entered a plea of guilty.[4] In virtue of
his plea of guilty, the trial court proceeded to satisfy itself of the voluntariness of the plea by propounding
questions to the accused to find out if he understood his plea and the legal consequence
thereof. Accused, assisted by counsel, reiterated his plea of guilty and the extra judicial confession he
executed before the police.
Nonetheless, the prosecution proceeded to present evidence to prove the presence of aggravating
circumstances. The accused on the other hand presented evidence proving the mitigating circumstances
that attended the commission of the crime.
The prosecution alleged that the aggravating circumstances of evident premeditation, cruelty,
nighttime, abuse of superior strength, disrespect to sex, and intoxication were present in the commission of
the crime. The accused, on the other hand, raised the attendance of the mitigating circumstances of
voluntary surrender, plea of guilty, mistaken identity and the alternative mitigating circumstance of
intoxication.
On September 2, 1998, the trial court after considering the aggravating and mitigating circumstances
attendant found the existence of the aggravating circumstances and appreciated only the mitigating
circumstance of plea of guilty that was offset by one of the aggravating circumstances. The trial court
then proceeded to rule on the appropriate penalty to be imposed on the accused. The trial court
rendered a decision, the dispositive portion of which reads:

49
WHEREFORE, the court finds the accused guilty beyond reasonable doubt of the crime of Murder, defined
and penalized under Article 248 of the Revised Penal Code, as amended by the Republic Act 7659,
otherwise known as the Death Penalty Law and is hereby sentenced to suffer the penalty of Death, to
indemnify the family of the victim in the amount of P60,000.00, and to pay damages in the amount of
P200,000.00 and cost .[5]

Hence, this review.[6]


Accused raises the following errors imputed to the trial court:
1. In finding that the aggravating circumstances of evident premeditation, cruelty, nighttime,
abuse of superior strength, sex and intoxication attended the commission of the crime charged;
and
2. In imposing the death penalty upon accused instead of reclusion perpetua.
According to the accused, the prosecution failed to prove the aggravating circumstances of evident
premeditation and other circumstances attending the commission of the crime.
We agree with the accused that the prosecution did not prove the aggravating circumstance of
evident premeditation. The prosecution failed to establish the following elements of this aggravating
circumstance: (a) the time when the accused determined to commit the crime, (b) an act manifestly
indicating that the accused clung to that determination, and (c) a lapse of time between the
determination and the execution sufficient to allow the accused to reflect upon the consequences of the
act.[7]
As to the aggravating circumstance of cruelty, although the accused stabbed the victim several
times, the same could not be considered as cruelty because there was no showing that it was intended
to prolong the suffering of the victim. For cruelty to be appreciated against the accused, it must be shown
that the accused, for his pleasure and satisfaction, caused the victim to suffer slowly and painfully as he
inflicted on him unnecessary physical and moral pain. The crime is aggravated because by deliberately
increasing the suffering of the victim the offender denotes sadism and consequently a marked degree of
malice and perversity. The mere fact of inflicting various successive wounds upon a person in order to
cause his death, no appreciable time intervening between the infliction of one (1) wound and that of
another to show that he had wanted to prolong the suffering of his victim, is not sufficient for taking this
aggravating circumstance into consideration.[8]
As to the aggravating circumstance of nighttime, the same could not be considered for the simple
reason that it was not specifically sought in the commission of the crime. Night-time becomes an
aggravating circumstance only when (1) it is specially sought by the offender; (2) the offender takes
advantage of it; or (3) it facilitates the commission of the crime by insuring the offender's immunity from
identification or capture.[9] In the case at bar, no evidence suggests that accused purposely sought the
cover of darkness to perpetrate the crime, or to conceal his identity.
The trial court erred in further appreciating the aggravating circumstance of abuse of superior
strength. Abuse of superior strength is absorbed in treachery, so that it can not be appreciated separately
as another aggravating circumstance.[10] Here, treachery qualified the offense to murder.
As to the aggravating circumstance of disregard of sex, the same could not be considered as it was
not shown that accused deliberately intended to offend or insult the sex of the victim, or showed manifest
disrespect for her womanhood.[11] In fact, the accused mistook the victim for a man.
Ordinarily, intoxication may be considered either aggravating or mitigating, depending upon the
circumstances attending the commission of the crime. Intoxication has the effect of decreasing the
penalty, if it is not habitual or subsequent to the plan to commit the contemplated crime; on the other
hand, when it is habitual or intentional, it is considered an aggravating circumstance. A person pleading

50
in toxication to mitigate penalty must present proof of having taken a quantity of alcoholic beverage prior
to the commission of the crime, sufficient to produce the effect of obfuscating reason. At the same time,
that person must show proof of not being a habitual drinker and not taking the alcoholic drink with the
intention to reinforce his resolve to commit the crime.[12]
Accused argues that in the absence of any of the aggravating circumstances alleged in the
information and considering that there was one mitigating circumstance attendant, that of plea of guilty,
the penalty imposable is not death but reclusion perpetua.
The Solicitor General agrees with the accused that the only aggravating circumstance present was
treachery which qualified the killing to murder and that there were two mitigating circumstances of plea
of guilty and intoxication, not habitual. The penalty shall be reclusion perpetua, not death, in accordance
with Article 63 in relation to Article 248 of the Revised Penal Code, as amended by Republic Act No. 6759.
We also award P50,000.00 as moral damages in keeping with current jurisprudence. Moral damages is
proper considering the mental anguish suffered by the heirs of the victim on account of her untimely and
gruesome death.[13]
WHEREFORE, the decision of the Regional Trial Court, Surigao del Sur, Branch 29, Bislig, in Criminal Case
No. 2026 convicting accused George Cortes y Ortega of murder is AFFIRMED with MODIFICATION as to the
penalty imposed. In lieu of the death penalty, the accused George Cortes y Ortega is hereby sentenced
to reclusion perpetua, with all the accessory penalties of the law, to indemnify the heirs of the victim in the
amount of fifty thousand pesos (P50,000.00) as death indemnity, and fifty thousand pesos (P50,000.00) as
moral damages and to pay the costs of suit.

People vs Regala
G.R. No. 130508
April 5, 2000
Criminal Case Digest

Facts:

On the night of September 11, 1995, at Barangay Bangon in Aroroy, Masbate, then 16-year old victim
Nerissa Tagala and her grandmother, Counselo Arevalo, were sleeping, when appellant Armando Regala
and his two other companions entered the formers house.

Appellant and his companions entered the house through the kitchen and went to the room of the victims
and poked at 8-inch gun on them, one after the other, and hogtied both of them. Armando raped Nerissa
in bed while her grandmother was hogtied on the floor. Later, she saw her grandmothers aparador being
opened where two rings, two wrist watches, and money were taken from the aparador. After raping her
in bed, Nerissa saw accused-appellant counting the money taken from the aparador. Thereafter, she was
brought to the kitchen, still hogtied and was raped again by the accused.

He was convicted in the lower court but accused-appellant appealed his criminal case at the Regional
Trial Court in Masbate. He questioned the sufficiency of the prosecutions evidence in identifying him as
one of the perpetrators of the crime charged. And based on medico-legal, Dr. Conchita Ulanday, a health
officer of Aroroy, testified herself that the complaining witness either voluntarily submitted to a sexual act
or was forced into one.

Issue:

(a) Whether additional rape committed in a crime of robbery be considered as an aggravating

51
circumstance?

Held:

On cross-examination, both Nerissa Tagala and Consuelo Arevalo, separately testified that they saw the
face of Regala, despite of no electricity at the commission of the crime, because he used a flashlight and
took off the mask he was wearing, and thus, they remembered him wearing an earring of his left ear, which
he was still wearing at the time of the police line-up inside the police station.

The trial court held that contradiction referred to a minor detail, cannot detract from the fact, that both
Nerissa and Consuelo positively identified the accused-appellant. As correctly pointed out by the
appellee, the victim was a 16-year old barrio lass, not exposed to the ways of the world and was not shown
to have any ill-motive to falsely implicate accused-appellant, who was a stranger. Hence, Dr. Ulandays
testimony does not support the contention of accused-appellant that the victim voluntarily submitted to
sexual advances of Regala.

The crime of robbery with rape was committed in 1995 when RA 7659 was already in force. Under Article
294 of the Revised Penal Code as amended, now provides, under paragraph 1 thereof: (1) The penalty of
reclusion perpetua to death, when for any reason of or on occasion of the robbery, the crime of homicide
shall have been committed, or when the robbery shall have been accompanied by rape or intentional
mutilation or arson.

In this case, the additional rape committed by herein accused-appellant should not be considered as
aggravating. The penalty of reclusion perpetua imposed by the trial court is proper. The judgment
convicting Armando Regala y Abriol guilty beyond reasonable doubt of the crime of Robbery with Rape,
where the victim is entitled to an additional award of P50,000.00 as civil indemnity.

People vs Jaime Jose y Gomez, et al.


Chester Cabalza recommends his visitors to please read the original & full text of the case cited. Xie xie!

People vs Jaime Jose y Gomez, et al.


G.R. No. L-28232
February 6, 1971

Facts:

On June 26, 1967, four (4) principal-accused Jaime Jose, Basilio Pineda Jr., alias Boy, Eduardo Aquino
Alias Eddie and Rogelio Caal; together with Wong Lay Pueng, Silverio Guanzon and Jessie Guion as
accomplices, conspired together, confederated with and mutually helped one another, then and there,
to willfully, unlawfully and feloniously, with lewd design to forcibly abduct Magdalena Maggie de la Riva,
25 years old and single, a movie actress by profession at the time of the incident, where the four principal
accused, by means of force and intimidation using a deadly weapon, have carnal knowledge of the
complainant against her will, and brought her to the Swanky Hotel in Pasay City, and hence committed
the crime of Forcible Abduction with Rape.

Wherefore, the court finds that the accomplices Pueng, Guanzon and Guion, on the ground that the
prosecution has failed to establish a prima facie case against them, the Motion to Dismiss filed for and in
their behalf is hereby granted, and the case dismissed against them.

(Facts of this case are too descriptive. Id rather not include much details on the scene of the crime to
protect the complainants repute).

52
Issue:

(a) What kind of rape was committed?

Held:

Undoubtedly, rape is that which is punishable by the penalty reclusion perpetua to death, under
paragraph 3, Article 335, as amended by Republic Act 4111 which took effect on June 20, 1964. Under the
law, rape is committed by having canal knowledge of a woman under any of the following circumstances:
(1) by using force and intimidation; (2) when the woman is deprived of reason and otherwise unconscious;
and (3) when the woman is under twelve years of age, even though neither of the circumstances
mentioned in the two next preceding paragraphs shall be present. The crime of rape shall be punished by
reclusion perpetua. Whenever the rape is committed the use of a deadly weapon or by two or more
persons, the penalty shall be reclusion perpetua to death.

As regards, therefore, the complex crime of forcible abduction with rape, the first of the crimes committed,
and the latter is the more serious; hence, pursuant to the provision of Art 48 of the RPC, the penalty
prescribed shall be imposed in its maximum period. Consequently, the appellants should suffer the extreme
penalty of death. In this regard, there is hardly any necessity to consider the attendance of aggravating
circumstances, for the same would not alter the nature of the penalty to be imposed.

However, said crime as attended with the following aggravating circumstances: (a) nighttime, appellants
having purposely sought such circumstance to facilitate the commission of these crimes; (b) abuse of
superior strength, the crime having been committed by the four appellants in conspiracy with one another;
(c) ignominy, since the appellants in ordering the complaint to exhibit to them her complete nakedness
for ten minutes before raping her, brought about a circumstance which tended to make the effects of the
crime more humiliating; and (d) the use of motor vehicle.

Of the three principal-appellants (Jose, Aquino and Caal), none of them may claim aggravating
circumstances has been offset by the mitigating circumstance. Appellant Pineda should, however, be
credited with the mitigating circumstance of voluntary plea of guilty, a factor which does not in the least
affect the nature of the proper penalties to be imposed, for the reason that there would still be three
aggravating circumstances remaining.

Insofar as the car used in the commission of the crime is concerned, the order of the court a quo for its
confiscation is hereby set aside; and whoever is in custody thereof is hereby ordered to deliver its possession
to intervenor Filipinas Investment & Finance Corporation in accordance with the judgment of the First
Instance of Manila in Civil Case No. 69993 thereof.

Before the actual promulgation of the decision, the Court received a formal manifestation on the part of
the Solicitor general to the effect that Rogelio Caal, one of the herein appellants, died in prison on
December 28, 1970. As a result, the case is dismissed as to him alone, and only insofar as his criminal liability
is concerned, with one-fourth (1/4) of the costs declared de officio.

Wherefore, the judgment under review is hereby modified as follows: Jaime G. Jose, Basilio Pineda, Jr., and
Eduardo P. Aquino are pronounced GUILTY of the complex crime of forcible abduction with rape, and
each and every one of them likewise convicted of three (3) of the crimes of rape. As a consequence
thereof, each of them is hereby sentenced to four (4) death penalties; all of them shall jointly and severally,
indemnify the complainant of the sum of P10,000.00 in each of the four crimes, or a total of 40,000.00; and
each shall pay one-fourth of the costs.

53
G.R. No. L-20183 June 30, 1966

THE PEOPLE OF THE PHILIPPINES, plaintiff and appellee,


vs.
EDUARDO BERDIDA Y INGUITO, ET AL., defendants.
EDUARDO BERDIDA Y INGUITO, LORETO SABERON Y CASAS, VICENTE ABERAS Y CORDERO and JESUS FELICIA
Y BALIDBID, defendants and appellants.

Senen S. Ceniza, Emilio G. Opinion and Agustin R. Romeras for defendants and appellants.
Office of the Solicitor General A. A. Alafriz, Acting Assistant Solicitor General I. C. Borromeo and Solicitor S.
C. Jacob for plaintiff and appellee.

PER CURIAM:

This is an automatic review of death sentence pursuant to the Rules of Court.1

On 10 May 1960, an information for frustrated murder2 of Antonio Maravilla and another information for
murder3 of Federico Caalete, were filed in the Court of First Instance of Manila. Said informations were
directed against the same eight accused: Eduardo Berdida y Inguito, Jesus Felicia y Balidbid, Vicente
Aberas y Cordero, Cristoto Mitilla y Paral, Demetrio Garin y Payos, Protacio Libres y Corona, Loreto Saberon
y Casas and Mario Mustrado y Sumaya.

After the defendants pleaded not guilty at their arraignment on 16 May 1960, the two cases were tried
jointly. Acting on a motion to dismiss filed by defendants Cristoto Mitilla and Mario Mustrado, after the
prosecution rested its case, the court dismissed the charges against Mario Mustrado, with costs de oficio.
After the trial, the Court of First Instance rendered on 27 July 1962 the decision now under review. Its
dispositive portion states:

In view of the foregoing considerations, the Court finds the defendants Eduardo Berdida, Loreto
Saberon, Vicente Aberas and Jesus Felicia guilty beyond reasonable doubt of the crime of murder.
This Court has in previous cases endeavored to avoid the imposition of the capital punishment. In
the case at bar, however, where the offenders, pretending to be police officers, kidnapped the
victims and mercilessly beat one of them to death, the Court finds no other alternative, in pursuance
to the mandate of the law, but to impose, as it hereby imposes upon the said defendants, the death
penalty, to indemnify jointly and severally the heirs of Federico Caalete in the sum of P4,000.00 and
to pay the costs. May God have mercy on their souls.

In Criminal Case No. 52338, above-said defendants are also hereby found guilty beyond reasonable
doubt of the crime of attempted murder and considering the aggravating circumstances present,
they are sentenced each to suffer a maximum penalty of TEN (10) YEARS of prision mayor and a
minimum of SIX (6) YEARS ofprision correccional, and to pay the costs, without prejudice on the part
of the complainant to institute a separate civil action for the recovery of damages.

The defendants Garin, Mitilla and Libres are hereby acquitted, in both cases, with costs de oficio,
and their immediate release is hereby ordered.

So ordered.

54
The records show the prosecution's evidence, as follows:

At about 10 o'clock in the evening of 7 May 1960, Antonio Maravilla, Federico Caalete, Virgilio Haban
and Pedrito Rapadas left the store of one Mang Terio at Mabuhay Street, North Harbor, Tondo, Manila,
and proceeded walking towards their homes. They were met on their way by Eduardo Berdida, Antonio
Louie, one Tiquio and one aliasIfugao, who identified themselves as detectives, told them not to move,
and pointed sharp and long bolos to them.4Antonio Maravilla and Federico Caalete raised their hands,
but Pedrito Rapadas and Virgilio Haban were able to run away. Antonio Louie then dealt a fist blow on
Antonio Maravilla. After that, the group took Antonio Maravilla and Federico Caalete along the rail
tracks, telling them that they had done something wrong.

At the end of the rail tracks, said group tied the hands of Antonio Maravilla and Federico Caalete. After
doing this, they dragged the two and took them to a place in Pier 8 at the North Harbor near Vicente
Aberas' house. In said place, there were others who joined the group, among them, Jesus Felicia, Loreto
Saberon and Vicente Aberas. At this point Eduardo Berdida told Antonio Maravilla and Federico Caalete
to dig their graves, but they refused. Arturo Macabebe, who also joined the group, took two sticks of
cigarettes and told Antonio Maravilla and Federico Caalete to smoke. Antonio Maravilla again refused.
Following said refusal, the victims were hit with a piece of wood. Eduardo Berdida and Jesus Felicia then
held Antonio Maravilla and Federico Caalete, respectively, by the hands and from behind. As they were
thus held, Vicente Aberas delivered fist blows on them, first on Antonio Maravilla, then on Federico
Caalete.5 Furthermore, Loreto Saberon also held Federico Caalete while others gave fist blows to the
latter.6 At about 1 o'clock in the morning of 8 May 1960, Antonio Maravilla lost consciousness, shortly after
hearing Loreto Saberon say that the group would cut off the ears of Antonio Maravilla and Federico
Caalete for appetizer or "pulutan".7

Antonio Maravilla's sister, Elizabeth, had meanwhile been informed by Virgilio Haban, one of those who
were able to run away, that her brother and Federico Caalete were taken by armed men. She therefore
went out with some companions in search of her brother. She asked the help of Patrolman Carlos Pili, who
was then at the corner of Kaguitingan and Lakandula Streets in front of Pier 6. Patrolmen Amado Santos
and Fabricante also joined them. As the other policemen took to separate directions, Patrolman Pili and
Elizabeth Maravilla went along Mabuhay Street. They came upon a group of men, between Piers 6 and 8,
who were hesitant to answer their inquiries. So they proceeded further, entering a small alley. As they went
on, Elizabeth found the shoes of her brother. So they continued until they met Vicente Aberas, stripped to
the waist, with bloodstains on his hands.8 Patrolman Pili detained him. Since somebody threatened them
should they proceed any further, Patrolman Pili and Elizabeth Maravilla went to Precinct 3, taking along
Vicente Aberas. Assistance from the Mobile Patrol was then requested. Accompanied by her neighbors
and more policemen, Elizabeth, together with Patrolman Pili, returned and went further to the interior of
Mabuhay Street. Finally, they came upon Federico Caalete and Antonio Maravilla, sprawled on the
ground, the former face down, the latter flat on his back. Federico Caalete was found dead. Antonio
Maravilla was alive, though his face was swollen, rendering him barely recognizable. Antonio Maravilla
was taken to the North General Hospital.

Patrolman Pili, meanwhile, went still further to the interior and saw, about 12 meters away from where they
found the victims, a group drinking liquor. At the approach of Patrolman Pili, about four men ran away,
leaving behind four men, namely, Loreto Saberon, Mario Mustrado, Cristoto Mitilla and Protacio Libres, the
last mentioned being then drunk and asleep on a bamboo bed.9 A Mobile Patrol car thereafter arrived
and apprehended them, except Libres. Patrolman Pili next went towards a house near Tagumpay Street
in which direction the others had fled. In said house, which was that of Crisanta Melgar, the patrolman
found some persons who pretended to be sleeping, namely, Demetrio Garin, Jesus Felicia and Eduardo
Berdida. Patrolman Pili brought them outside and they were taken by the Mobile Patrol to the Detective
Bureau.

55
Furthermore, the body of Federico Caalete was examined at the scene where it was found by officers of
the Mobile Patrol. Detective Bureau agents likewise went to said place. Finding bloodstains near an a alley
to Tagumpay Street, they went to a house thereat and found Protacio Libres sleeping on a bamboo bed.
Said detectives took Libres to the headquarters.

At the police station, all the apprehended suspects were made to mingle with other persons. Antonio
Maravilla, who was fetched to point out therefrom the persons who attacked him and Federico Caalete
identified Eduardo Berdida, Vicente Aberas, Loreto Saberon and Jesus Felicia.

An autopsy was made on 8 May 1960 on the body of Federico Caalete by Dr. Luis Larion, Medical
Examiner of the Manila Police Department. The post mortem findings in his report are as follows: (Exh. M):

CENTRAL NERVOUS SYSTEM:

Hemorrhage extensive, subarachnoid brain.

CARDIOVASCULAR SYSTEM:

Laceration, blood vessels, brain and spleen.

RESPIRATORY SYSTEM:

Contusion, posterior lung, bilateral.

Congestion, lungs, bilateral.

GASTROINTESTINAL SYSTEM:

About 150 cc. partially digested rice meal with slight alcoholic odor.

Hemoperitoneum about 100 cc. blood, abdominal cavity.

SPLEEN: Maceration spleen.

PANCREAS: Contusion, hemorrhagic, pancreas.

BONES AND JOINTS:

Fracture-separation, left parieto-occipital and right fronto-temporal skull.

MISCELLANEOUS:

Wound, stab, non-penetrating, 1.3 x 0.5 cm. x 1.5 cm. deep, right lumbar region.

Wound, lacerated, 3 x 0.5 cm. occipital region.

Wound, lacerated, 2.5 cm. x 1.5 cm. x 1 cm. deep, non-penetrating, left abdomen.

Hematoma, frontal, right; left, parieto-occipital, and occipital, scalp, head.

56
Contusion, multiple, left forehead; left lower eyelids; left face; nose; lower lip; left lateral neck;
posterior neck; left shoulder; left and right posterior chest.

Contused abrasion, anterior left lower chest and right abdomen.

CAUSE OF DEATH:

Shock and hemorrhage due to traumatic fracture of the skull with maceration of spleen,
contusion of the lungs and extensive subarachnoid hemorrhages in the brain.

Antonio Maravilla, as shown in the medico-legal certificate of Dr. Cumalinga Espinosa of the North General
Hospital (Exh. R), sustained these injuries:

Contusion with abrasion, and periorbital hematoma, eye right.

Contusion upper and lower lip.

Contusion 2" mental region.

Contusion with slight hematoma, malar right, and mandible bilateral.

Abrasion, 3", lateral neck left.

Abrasion, 2" #2 level of the 10th rib right, along the MCL.

For the defense of herein appellants, the following evidence was presented to establish alibi:

Sometime between 7 and 8 o'clock in the evening of 7 May 1960 Crisanta Melgar was filling drums with
water in her house at 1205 Tagumpay Street, Tondo, Manila. Shortly thereafter, Eduardo Berdida, Loreto
Saberon and Jesus Felicia arrived. Since her husband was on night duty and her brother- in-law was ill,
Crisanta Melgar asked the three to remain and help her fill up the drums with water, intending to sell the
same the next morning. Said defendants consented and for some time helped Crisanta fill the drums with
water. At about 9 o'clock in the evening, however, said defendants went to sleep in the ground floor of
Crisanta's new house, still under construction, adjacent to the house aforementioned. At about midnight
a policeman and someone in civilian clothes knocked at the door and inquired from Crisanta if there were
three persons sleeping in her house. She said yes, and opened the door. The policeman then told Crisanta
that a dead man was found near their place. The one in civilian attire went to the back of the house.
Crisanta told the policeman she knew nothing of any incident and that the three men had been in her
house for some time. She then awoke the defendants Berdida, Saberon and Felicia. The policeman told
them to stand up and the man in civilian was asked if they were the ones involved. Said man looked at
the defendants and replied in the negative. The policeman and the civilian then left and the defendants
went back to sleep. After a while, Crisanta, who was restless and could not sleep, went down, awoke the
defendants, and told them that it was better for them to leave. So, the said defendants left, but a
policeman stopped them at Tagumpay Street and took them to the police headquarters.

As to the defendant Vicente Aberas, his defense of alibi is as follows:

In the evening of 7 May 1960, he was on board the fishing boat "Don Paulino." At about 10:30 o'clock in
the evening, after unloading their catch of fish, he left for home, bringing with him a tulingan fish. Juan, a
co-worker of his, invited him to drink beer in a store near Pier 8. For some time they stayed there, then he
left for home. On the way he met five men beating up somebody. Approaching them, he asked them to

57
have pity on the man and not to beat him. Someone in the group, armed with a club, warned him not to
interfere, so, becoming afraid, he left. In reaching home, he took off his shirt, cut the fish he brought with
him in half, lengthwise, and took one of the halves to the house of Emiliano Retone, another co-worker of
his, who did not report for work that day. Retone invited him to drink gin. After drinking, he headed for
home, but on his way he met two policemen and a woman. After being asked where he came from,
which he answered, and whether he had seen a fight, to which he said yes, he was taken to Precinct 3.

Appellants would, first of all, assail Antonio Maravilla's testimony identifying them as the assailants, for the
reason that he lost consciousness, and, therefore, could not be relied upon to make said identification.
Appellants would further insist on their defense of alibi. Antonio Maravilla, it is true, lost consciousness' at
about 1 o'clock in the morning of 8 May 1960. It is however equally true that before his sense faded out he
saw herein appellants perform their atrocities on himself as well as on Federico Caalete. It cannot
therefore be doubted that he made no mistake in pointing out to herein appellants as definitely among
their assailants. This he did, not only at the police station but also in open court during the trial. It is
furthermore not disputed by defendants-appellants that Antonio Maravilla has no reason or motive to
falsely accuse them of murder and attempted murder. The positive identification he made must therefore
be given credence.

It follows that the defense of alibi cannot be sustained. The rule is settled, to the point of being trite, that
the defense of alibi is worthless in the face of positive identification by prosecution witnesses, pointing to
the accused as participants in the crime. 10

The trial court, moreover, found the above-related defenses of alibi not credible. For, according to said
court, if defendants Berdida, Felicia and Saberon really went to help Crisanta Melgar, their provincemate,
fill drums with water at her house, it is rather unusual that they went to sleep at about 9 o'clock in the
evening. Furthermore, the policeman who inquired about persons sleeping in Crisanta Melgar's house
strangely knew their number, that is, three persons. And, finally, it is unbelievable that said policeman did
not take them to the headquarters for identification by Antonio Maravilla himself.1wph1.t

And, with respect to the defendant-appellant Vicente Aberas, the trial court found it too surprising to
believe that he went to such lengths of amiability, as to go, shirtless at that, to his friend Retone, at an
unholy hour, to share with him one-half of his tulingan fish. No previous agreement, or urgent need for such
an act obtained. It could have waited for the next morning, especially since, having allegedly come from
work, defendant Aberas must have been tired.

As this Court stated in People vs. Constante, L-14639, December 28, 1964, the defense of alibi is an issue of
fact that hinges on credibility; that the credibility of an alibi depends so much on the credibility of the
witnesses who seek to establish it; and that, in this inspect, the relative weight which the trial judge assigns
to the testimony of said witnesses must, unless patently and clearly inconsistent with the evidence on
record, be accepted. For, as is well recognized, his proximate contact with those who take to the witness
chair places him, compared to appellate Justices, in the more competent position to discriminate be
between the true and the false.

And in the present appeal, we find no warrant to depart from the lower court's finding on defendants-
appellants' defense of alibi.

It is also contended by appellants that the aggravating circumstances of nighttime, abuse of superior
strength, and evident premeditation should not be appreciated in fixing the penalty. Appellants would
argue that nighttime was not purposely sought to facilitate the offense or to afford impunity. At any rate,
they would further argue, nighttime as well as abuse of superior strength are deemed absorbed in
treachery. As to evident premeditation, they aver that the premeditation, if any, is not evident, for lack of

58
sufficient lapse of time between the execution of the offense and a previous showing of intent to commit
it, so as to show that the offenders clung to their determination to commit the crime.

The presence of one generic aggravating circumstance, apart from the qualifying circumstance of
treachery, suffices to fix the penalty for murder at the extreme punishment of death. For there is no
mitigating circumstance in the present case. From the facts and evidence of record in this case, it is clear
that appellants took advantage of nighttime in committing the felonies charged. For it appears that to
carry out a sentence they had pronounced upon Antonio Maravilla and Federico Caalete for the death
of one Pabling, they had evidently chosen to execute their victims under cover of darkness, at the dead
of night, when the neighborhood was asleep. Inasmuch as the treachery consisted in the fact that the
victims' hands were tied at the time they were beaten, the circumstance of nighttime is not absorbed in
treachery, but can be perceived distinctly therefrom, since the treachery rests upon an independent
factual basis. A special case therefore is present to which the rule that nighttime is absorbed in treachery
does no apply. 11

In addition, the presence of evident premeditation is likewise borne out by the record. For the victims were
told at the start, when they were taken captives, that they had done something wrong, that they were the
ones who stabbed and killed one Pabling, and that for this reason they were to go with the group (T.s.n.,
10 October 1960, pp. 20, 22; Exh. D). Not only that; the victims were then taken to a spot where they were
ordered to dig their graves. The assailants were previously armed with deadly weapons, and their assault
was a concerted and group action. From the time of apprehension of the victims, About 10 o'clock in the
evening, to the time Antonio Maravilla lost consciousness, about 1 o'clock early the following morning, is
sufficient time for the offenders to meditate and reflect on the consequences of their act.

In People vs. Lopez, 69 Phil. 298, this Court found the aggravating circumstance of evident premeditation
present, in view of the repeated statements of the defendants that the hour of reckoning of the victim
would arrive, the existing enmity between them, the fact that they were previously armed with deadly
weapons, and the fact that the aggression was simultaneous and continuous until the deceased was left
unconscious on the ground. And in People vs. Lazada, 70 Phil. 525, four hours was held sufficient lapse of
time for purposes of the presence of evident premeditation. Furthermore, sufficient lapse of time in this
regard is not simply a matter of the precise number of hours, but of the reasonable opportunity, under the
situation and circumstances, to ponder and reflect upon the consequences. In the present case, we find
the facts and circumstances obtaining sufficient to support the trial court's finding of the attendance of
evident premeditation.

Following previous instances, the indemnity to the heirs of the deceased in this case should be increased
to P6,000. 12

Anent the attempted murder case, no appeal therefrom was taken. The record shows that defendants
perfected no appeal from the judgment below. The present automatic review is limited only to the murder
case in which the death penalty was imposed. It was only because of the joint trial that the record of the
attempted murder case was likewise elevated herein. Since no appeal was taken in the attempted murder
case, the judgment with respect thereto has become final. It therefore cannot now be reviewed herein,
as some of the appellants would ask. And defendants-appellants, who are detained, should accordingly
be deemed to have started serving their respective sentence in said attempted murder case from the
time the decision of the trial court became final as to said case.

Wherefore, the death penalty imposed on defendants-appellants is hereby affirmed, and the indemnity
to the heirs of Federico Caalete is hereby increased from P4,000 to P6,000, with costs. So ordered.

59

S-ar putea să vă placă și